You are on page 1of 55

SEPTEMBER 2016

Vol. 37 No. 9
www.pedsinreview.org

Acid-Base Disorders
Hsu, Lakhani, Wilhelm

Evaluation and Initial


Management of Hypopituitarism
Pierce, Madison

Chronic and Recurrent


Abdominal Pain
Hyman

ONLINE

Visual Diagnosis:
Spoon Nails and Short, Brittle
Hair in a 3-year-old Boy
Sorenson, Tom
What’s your
specialty?

PREP (Pediatrics Review and Education Program) is produced yearly by AAP topic experts
to keep you current and prepared in general pediatrics and 12 areas of specialty care:
General Pediatrics | Adolescent Medicine | Cardiology | Critical Care | Developmental-Behavioral | Emergency Medicine | Endocrinology
Gastroenterology | Hematology-Oncology | Infectious Diseases | Neonatal-Perinatal Medicine | Nephrology | Pulmonology

RT U U
Learn why thousands of pediatric professionals subscribe to
PA U PREP Self-Assessments are approved by the
C

UU

2
U MO

American Board of Pediatrics for MOC Part 2:


UUUU

PREP every year for continuing education and board review.


UU

UU UUU
U
U
Lifelong Learning and Self-Assessment points.

Start your subscription today


at shop.aap.org/prep.
contents

Pediatrics in Review ®
Vol. 37 No. 9 September 2016

Editor-in-Chief: Joseph A. Zenel, Sioux Falls, SD


Deputy Editor: Hugh D. Allen, Houston, TX
Associate Editor, Index of Suspicion: Philip R. Fischer,
ARTICLES Rochester, MN
Associate Editor, Index of Suspicion: Deepak M. Kamat,
361 Acid-Base Disorders Detroit, MI
Associate Editor, Visual Diagnosis: Mark F. Weems,
Benson S. Hsu, Saquib A. Lakhani, Michael Wilhelm Memphis, TN
Associate Editor, In Brief: Henry M. Adam, Bronx, NY
370 Evaluation and Initial Management of Associate Editor, In Brief: Janet Serwint, Baltimore, MD
Hypopituitarism Associate Editor, CME: Rani Gereige, Miami, FL
Editorial Fellow: Aamir Jeewa, Houston, TX
Melinda Pierce, Lisa Madison Early Career Physician: Heather Campbell, Washington, DC
Editor Emeritus: Lawrence F. Nazarian, Rochester, NY
377 Chronic and Recurrent Abdominal Pain Founding Editor: Robert J. Haggerty, Canandaigua, NY
Paul E. Hyman Managing Editor: Luann Zanzola
Publications Editor: Sara Strand
INDEX OF SUSPICION Medical Copyediting: Deborah K. Kuhlman

391 Case 1: Constipation, Irritability, and Poor EDITORIAL BOARD


Robert D. Baker, Buffalo, NY
Feeding in 2-month-old Boy Peter F. Belamarich, Bronx, NY
Nidhi A. Shah, Henry C. Lee Eyal Ben-Isaac, Los Angeles, CA
Theresa Auld Bingemann, Rochester, NY
394 Case 2: Acute Respiratory Distress and Facial Stephen E. Dolgin, New Hyde Park, NY
Lynn Garfunkel, Rochester, NY
Petechiae in a 4-month-old Boy Rani Gereige, Miami, FL
Alpa Patel Shah, Mary Lu Angelilli Nupur Gupta, Boston, MA
Gregory A. Hale, St. Petersburg, FL
397 Case 3: Sore Throat and Fever in a Thomas C. Havranek, Bronx, NY
Jacob Hen, Bridgeport, CT
4-year-old Boy Jeffrey D. Hord, Akron, OH
Katsuaki Kojima, Melissa Rosenberg, B. Keith English Neal S. LeLeiko, Providence, RI
Michael Macknin, Cleveland, OH
399 Case 4: Two Black Eschars in a 6-year-old Girl Susan Massengill, Charlotte, NC
Carrie A. Phillipi, Portland, OR
Zachary Graff, Geronimo Mendoza, Apurv Barche, Curtis Turner Peter Pizzutillo, Philadelphia, PA
Mobeen Rathore, Jacksonville, FL
IN BRIEF Jennifer S. Read, Rockville, MD
402 Pulse Oximetry and the Neonate E. Steve Roach, Columbus, OH
Sarah E. Shea, Halifax, Nova Scotia
Philip Roth Andrew Sirotnak, Denver, CO
Miriam Weinstein, Toronto, ON
405 CME Quiz Corrections
PUBLISHER: American Academy of Pediatrics
ONLINE Mark Grimes, Director, Department of Publishing
Joseph Puskarz, Director, Division of Journal Publishing
e38 Visual Diagnosis: Spoon Nails and Short,
Brittle Hair in a 3-year-old Boy Pediatrics in Review offers 36 CME articles per year. A maximum
of one AMA PRA Category 1 Credit TM is earned after achieving a
Eric P. Sorensen, Wynnis L. Tom 60% score on each designated quiz.
CME STATEMENTS:
The American Academy of Pediatrics (AAP) is accredited by
®
Pediatrics in Review (ISSN 0191-9601) is owned and controlled by the American Academy of Pediatrics. the Accreditation Council for Continuing Medical Education
It is published monthly by the American Academy of Pediatrics, 141 Northwest Point Blvd., Elk Grove Village, IL 60007-1098. (ACCME) to provide continuing medical education for
Statements and opinions expressed in Pediatrics in Review® are those of the authors and not necessarily those of the American physicians.
Academy of Pediatrics or its Committees. Recommendations included in this publication do not indicate an exclusive course of The AAP designates this journal-based CME activity for a
treatment or serve as a standard of medical care. maximum of 1.00 AMA PRA Category 1 Credit TM. Physicians
Subscription price for 2016 for print and online/online only: AAP/CPS Member $204/$156; AAP National Affiliate Member $160/$110; should claim only the credit commensurate with the extent
Nonmember $255/$198; Allied Health or In-training $190/$128. Institutions call for pricing (866-843-2271). For overseas delivery, add $120. of their participation in the activity.
Current single issue price is $10 domestic, $12 international. Replacement issues must be claimed within 6 months from the date of issue
and are limited to three per calendar year. Periodicals postage paid at ARLINGTON HEIGHTS, ILLINOIS and at additional mailing offices. This activity is acceptable for a maximum of 1.00 AAP credit.
© AMERICAN ACADEMY OF PEDIATRICS, 2016. These credits can be applied toward the AAP CME/CPD*
All rights reserved. Award available to Fellows and Candidate Members of the
Printed in USA. No part may be duplicated or reproduced without permission of the American Academy of Pediatrics. AAP.
POSTMASTER: Send address changes to PEDIATRICS IN REVIEW®, American Academy of Pediatrics Customer Service Center, 141 The American Academy of Physician Assistants accepts
Northwest Point Blvd., Elk Grove Village, IL 60007-1098. certificates of participation for educational activities certified
Pediatrics in Review® Print Issue Editorial Board Disclosures for AMA PRA Category 1 Credit TM from organizations accredited
The American Academy of Pediatrics (AAP) Policy on Disclosure of Financial Relationships and Resolution of Conflicts of Interest for by ACCME. Physician assistants may receive a maximum of 1.00
AAP CME Activities is designed to ensure quality, objective, balanced, and scientifically rigorous AAP CME activities by identifying hour of Category 1 credit for completing this program.
and resolving all potential conflicts of interest before the confirmation of service of those in a position to influence and/or control
CME content. All individuals in a position to influence and/or control the content of AAP CME activities are required to disclose to the This program is accredited for 1.00 NAPNAP CE contact
AAP and subsequently to learners that the individual either has no relevant financial relationships or any financial relationships with hour; pharmacology (Rx) and psychopharmacology contact
the manufacturer(s) of any commercial product(s) and/or provider(s) of commercial services discussed in CME activities. Commercial hours to be determined per the National Association of
interest is defined as any entity producing, marketing, reselling or distributing health-care goods or services consumed by, or used Pediatric Nurse Practitioners (NAPNAP) Continuing Education
on, patients. Guidelines.
Each of the editorial board members, reviewers, question writers, PREP Coordinating Committee members and staff has disclosed, if It has been established that each CME activity will take the
applicable, that the CME content he/she edits/writes/reviews may include discussion/reference to generic pharmaceuticals, off-label learner approximately 1 hour to complete.
pharmaceutical use, investigational therapies, brand names, and manufacturers. None of the editors, board members, reviewers,
question writers, PREP Coordinating Committee members, or staff has any relevant financial relationships to disclose, unless noted *Continuing Professional Development
below. The AAP has taken steps to resolve any potential conflicts of interest. How to complete this activity:
Disclosures
Pediatrics in Review can be accessed and reviewed in print or
• Robert Baker, MD, PhD, disclosed he has an author relationship with AstraZeneca.
online at http://pedsinreview.aappublications.org. Learners
• Lynn Garfunkel, MD, FAAP, disclosed that her family member is an employee of Philips Healthcare.
can claim credit monthly online upon completion of each CME
• Nupur Gupta, MD, MPH, disclosed she has a financial relationship with Springer US as co-editor for MassGeneral Hospital
article. The deadline for completing this activity is December
for Children Handbook of Pediatric Global Health.
31, 2018. Credit will be recorded in the year in which it is
• Gregory A. Hale, MD, disclosed that he is on the board of directors of Make A Wish, Suncoast Chapter, and the Sickle Cell
Disease Association of America, North Pinellas Chapter; and that he has research grants with Hyundai Hope on Wheels and submitted. It is estimated that it will take approximately 1 hour
with the V Foundation for Cancer Research. to complete each CME article. This activity is not considered
• Miriam Weinstein, MD, has disclosed she is on the advisory board of Tribute Pharmaceuticals; on the advisory board and has to have been completed until the learner documents
a consensus meeting/author relationship with Pierre Fabre; is a paid consultant and advisory board member for Johnson & participation in that activity to the provider via online
Johnson and Valeant; on the advisory board of Valeo Pharma Inc.; is a paid consultant, advisory board member, and receives submission of answers. Course evaluations are online.
honoraria and grants from LaRoche-Posay; and receives honoraria for lectures from Galderma and Pediapharm.
The journal extends special thanks to the following question writers and reviewers who contributed to this issue:
• Amika Adhikari, MD
• James Wilde, MD
• Catherine Wiley, MD
• H. Stephen Williams, MD Answer Key appears on page 405.
One

lucky
of the He survived
meningococcal disease.
Not everyone does.

ones
While rare, once meningococcal disease strikes, it can cause death in
10%-15% of patients in as few as 24 hours. Some patients consider
themselves lucky to have survived. However, 20% of survivors suffer
permanent consequences, including lost limbs, hearing loss, kidney
failure, and neurologic damage.1-5

Adolescent patients need 2 vaccines to help protect against the


5 vaccine-preventable serogroups—A, C, W-135, Y, and B. 1

To learn more, visit the-luckyones.com.

References: 1. Preteens, teens need meningococcal vaccine. Centers for Disease Control
and Prevention website. http://www.cdc.gov/features/meningococcal/. Updated October 22,
2015. Accessed April 8, 2016. 2. Thompson MJ, Ninis N, Perera R, et al. Clinical recognition
of meningococcal disease in children and adolescents. Lancet. 2006;367(9508):397-403.
3. Meningococcal disease. In: Hamborsky J, Kroger A, Wolfe S, eds. 13th ed. Epidemiology
and Prevention of Vaccine-Preventable Diseases. Washington, DC: Public Health Foundation;
2015. http://www.cdc.gov/vaccines/pubs/pinkbook/mening.html. Accessed March 11, 2016.
4. Slack R, Hawkins KC, Gilhooley L, et al. Long-term outcome of meningococcal sepsis-
associated acute renal failure. Pediatr Crit Care Med. 2005;6(4):477-479. 5. Vyse A, Anonychuk
A, Jäkel A, et al. The burden and impact of severe and long-term sequelae of meningococcal
disease. Expert Rev Anti Infect Ther. 2013;11(6):597-604.

The person depicted is a model used for illustrative purposes only.

© 2016 GSK group of companies. All rights reserved. Printed in USA. 704214R0 May 2016
One AAP Resource.
Advice From Thousands of Experts.
đ Red Book® Online: Including the 30th edition of Red Book®
in English and Spanish
đ Bright Futures Guidelines: All the visit forms and tools
đ Point-of-Care Quick Reference topics: More than 250
and growing
đ The Textbook of Pediatric Care: Including more
chapters from the second edition as they become
available
đ Pediatric Patient Education™: Hundreds of handouts in
English and Spanish pediatriccare.solutions.aap.org
supported by

The American Academy of Pediatrics is the exclusive owner of the Pediatric Care Online™ Web site and content.
Mead Johnson Nutrition has no control over or responsibility for the Pediatric Care Online™ Web site or content.

SAVE $75
with special package offer!
Order AAP Textbook of Pediatric Care, 2nd Edition,
and a 1-year subscription to Pediatric Care Online™ .
ĀĉĂą
Price: ĸąĈĉċĊĆ ĸąĀă Member Price: $402.95 $327

NEW 2nd Edition!


American Academy of Pediatrics
Textbook of Pediatric Care, 2nd Edition
CHAPTER 208 t Wheezing P6-503
The landmark guide to pediatric
medicine—updated and streamlined
Figure 208-8 Image from fiberoptic bronchoscopy.
Fiberoptic bronchoscopy demonstrates complete cartilagi-
nous rings causing severe obstruction of the trachea.

Figure 208-9 Three-dimensional reconstruction of


for today’s clinicians and students
computed tomographic image. Severe circumferential
narrowing of the trachea starting at the thoracic inlet
and extending to the main-stem bronchi.

Highlighting the numerous advances across the full span of pediatrics,


Includes A B

the new second edition of AAP Textbook of Pediatric Care provides a


free eBook C D
complete update of this premier clinical reference, including signs and
access!
Figure 208-10 Images from rigid bronchoscopy. A, Epiglottis. B, Severe stenosis secondary to a tracheal web. C and D,
After laser resection of the stenosis. (Courtesy of Sanjay Parikh, MD.)

symptoms, behavioral health, care of healthy and high-risk infants,


WHEN TO REFER
t 1PPS XFJHIU HBJO PS HSPXUI BTTPDJBUFE XJUI
t 1FSTJTUFOU PS SFDVSSFOU XIFF[JOH JO BO JOGBOU DISPOJDPSSFDVSSFOUXIFF[JOH
ZPVOHFSUIBOZFBS t 3FQFBUFEIPTQJUBMJ[BUJPOPSNVMUJQMFDPVSTFTPG
t "QQBSFOU QBSBEPYJDBM SFTQPOTF UP CSPODIPEJ PSBMDPSUJDPTUFSPJET
MBUPST t 1FSTJTUFOUBTZNNFUSJDXIFF[JOH

adolescent health, critical situations, practice management, ethical and


legal concerns, and much more.
Thomas K. McInerny, MD, FAAP; Hardcover with eBook Access, June 2016—3,192 pages
Henry M. Adam, MD, FAAP; MA0773
++'ƫ ƫĊĈĉġāġĆĉāāĀġĊććġĆƫđƫ!++'ƫ ƫĊĈĉġāġćāĀĀĂġĀąĈġă
Deborah E. Campbell, MD, FAAP; Price: $179.95 Member Price: $143.95
Thomas G. DeWitt, MD, FAAP;
Jane Meschan Foy, MD, FAAP; and
Deepak M. Kamat, MD, PhD, FAAP

Find out more at shop.aap.org


AD244 0716
Acid-Base Disorders
Benson S. Hsu, MD, MBA,* Saquib A. Lakhani, MD,† Michael Wilhelm, MD‡
*Pediatric Critical Care, University of South Dakota, Sanford School of Medicine, Sioux Falls, SD.

Pediatric Critical Care, Yale School of Medicine, New Haven, CT.

Pediatric Critical Care, University of Wisconsin School of Medicine and Public Health, Madison, WI.

Education Gap
To treat critically ill children, a physician must have a clear understanding
of acid-base balance.

Objectives After completing this article, readers should be able to:

1. Describe the mechanisms regulating acid-base physiology and identify


blood gas abnormalities associated with an acid-base imbalance.
2. Recognize the differential diagnosis and clinical and laboratory
features associated with metabolic acidosis and metabolic alkalosis as
well as how to manage each appropriately.
3. Calculate an anion gap and formulate a differential diagnosis
associated with various anion gap values.
4. Identify factors contributing to compensatory changes associated with
primary metabolic and respiratory acidoses and alkaloses.

INTRODUCTION

The body’s ability to maintain acid-base homeostasis is based on a complex set of


interactions between the respiratory and metabolic systems. This article reviews
normal acid-base physiology and examines disorders of acid-base imbalances, first
within a primary metabolic cause and then within a primary respiratory cause.
Covering the complex nuances of acid-base control within a limited-scope
review article is impossible. Thus, this article focuses on the traditional model
based on the Henderson-Hasselbalch equation rather than the strong ion (or
Stewart) model, which explores the difference between all the dissociated cations
and anions. Using the traditional model, the authors explore the various meta-
bolic and respiratory disturbances while addressing the implications of the anion
gap on metabolic acidoses.
AUTHOR DISCLOSURE Drs Hsu and Lakhani
have disclosed no financial relationships
relevant to this article. Dr Wilhelm has
REGULATION OF ACID-BASE
disclosed that he holds stock in Inovio
Pharmaceuticals. This commentary does not
The Henderson-Hasselbalch Equation
contain discussion of an unapproved/
investigative use of a commercial product or Homeostatic control of acid-base balance is critical for all metabolic and phys-
device. iologic functions of the human body. The Henderson-Hasselbalch equation

Vol. 37 No. 9 SEPTEMBER 2016 361


describes the relationship between pH and the bicarbonate ranges and not independently examine any of the variables.
buffering system (the predominant buffering system in For example, a blood gas with a high PCO2 may reflect a normal
plasma) to establish this homeostasis: pH and should be interpreted as nonacidotic (Table 1).

The Anion Gap


CO2 þ H2 O4H2 CO3 4Hþ þ HCO
3
Use of only the Henderson-Hasselbalch equation is insuf-
 
pH ¼ pK þ log HCO =H CO ficient to describe a patient’s metabolic acid-base state com-
3 2 3
pletely. The anion gap further describes the interactions of
the measured positive charges (cations) and negative charges
When accounting for H2CO3, the modified equation
(anions) to the unmeasured charged particles. The anion gap
becomes:
  equation is based on the understanding that the cations in the
pH ¼ pK þ log HCO
3 =½0:03∗Pco2  plasma balance the anions in the plasma at equilibrium.
The “measureable” positive and negative charges in the
Examination of the modified equation reveals the impor- serum refer to those measured with a standard electrolyte
tance of bicarbonate ion (HCO 3 ) and dissolved carbon
panel. To calculate the anion gap, positive charges include
dioxide (PCO2) in the determination of pH. The pK is the sodium (Naþ) and potassium (Kþ) while negative charges
pH at which the bicarbonate ion (HCO include bicarbonate (HCO 
3 ) and chloride (Cl ). In normal
3 ) and carbon
dioxide (CO2) are equal. This value is approximately conditions, the measured cations exceed the measured
6.35. The 0.03 constant is used to describe the PCO2 anions (the normal anion gap), which is predominantly
solubility. When changes occur in the pH due to PCO2 accounted for by the serum proteins. This normal value
changes, the predominant system involved is the respira- for the anion gap ranges from 12 to 20 mEq/L (12–20
tory system. When changes in pH occur due to changes in mmol/L) when the Kþ concentration is included and 8 to
HCO 3 , the predominant system involved is the metabolic
16 mEq/L (8-16 mmol/L) when it is not included.
system. However, the system favors the HCO
   
3 at physiologic 
Anion  Gap ¼ Naþ þ Kþ  HCO
3 þ Cl
pH, therefore, the buffering ability of the metabolic system
is dependent on the body’s ability to eliminate CO2 through
In the setting of low serum protein, such as hypoalbumi-
the respiratory system. Thus, it is evident that two comple-
nemia, the normal unmeasured anions are decreased and
mentary systems, respiratory and metabolic, are used to
the anion gap narrows. Thus, with critical illness when
describe changes to the body’s pH.
protein concentrations are often low, an elevated concen-
Clinically, the acid-base state is normally determined by
tration of unmeasured anions can frequently be masked
a blood gas sample. Although the gold standard remains
by an apparently normal (falsely low) anion gap. Conse-
arterial blood gas measurement, use of the venous or
quently, the combination of the modified Henderson-
capillary blood gas sample is prevalent within the pediatric
Hasselbalch equation and the anion gap calculation begins
population due to the relative ease in obtaining these sam-
to illustrate an individual patient’s acid-base balance.
ples. Although minor differences exist within pH and PCO2
among the different blood gas sample types, these differ-
ences can be accounted for during interpretation by assum-
ing expected slight increases in PCO2 and decreases in pH
TABLE 1. General Reference Ranges for Arterial
for venous samples. As expected due to the location of the
blood draw, venous blood gas samples are unreliable for PO2
and Venous Blood Gases
measurements. Because capillary blood gradually transitions ARTERIAL VENOUS
between arterial and venous states, the PCO2 and pH norma-
pH 7.38 – 7.42 7.36 – 7.38
tive values often are between the arterial and venous norma-
tive values. Of note, a blood gas machine measures the pH PO2 (mm Hg) 80 – 100 30 – 50
and the partial pressure of the gases, but the HCO 3 ion PCO2 (mm Hg) 38 – 42 43 – 48
concentration is a calculated value. HCO 22 – 24 25 – 26
3 (mmol/L)
In examining blood gases, acidosis occurs when the pH
value is lower than normal. In contrast, alkalosis occurs HCO 3 ¼bicarbonate, PCO2¼partial pressure of carbon dioxide, PO2¼ partial
pressure of oxygen. Reference normal values are laboratory-dependent
when the pH value is higher than normal. It is important to and may vary due to differing techniques.
characterize pH in context of PCO2 and HCO 3 reference

362 Pediatrics in Review


The human body has a natural inclination toward a desired Extending from this initial assessment, additional clues from
equilibrium, which accounts for the common finding of results of blood gas analysis and a basic metabolic panel reflect-
compensatory changes in the opposite direction. Within set- ing the anion gap can help discern a primary metabolic versus
tings of respiratory derangement leading to changes in the respiratory cause. Determining the acute or chronic nature of the
pH, the compensatory mechanism occurs within the meta- compensation adds additional diagnostic information.
bolic system. For respiratory acidosis (commonly arising due
to an increase of PCO2), the body compensates by creating
METABOLIC ACIDOSIS
metabolic alkalosis (from a retention of HCO 3 ). In contrast,
for respiratory alkalosis (commonly arising from a decrease A combination of the anion gap calculation and knowledge of
of PCO2), the body compensates by creating metabolic acidosis the pH state within the body is required to describe metabolic
(from a loss of HCO 3 ) (Table 2). Metabolic compensation to acidosis. The regulation of the acid-base state, as noted by the
respiratory derangements often takes hours to days to estab- Henderson-Hasselbalch equation, is based on the buffering
lish. In comparison, compensatory mechanisms within the effect of HCO 3 . However, from an electroneutrality view-
respiratory system for primary metabolic derangements gen- point, use of the anion gap calculation can further differentiate
erally occur more rapidly, often over minutes to hours. causes for the metabolic acidosis. Because reference values in
the calculation of unmeasured anions or anion gap vary
among laboratories, clinicians must be aware of normal values
ASSESSMENT AND DETERMINATION OF
for the laboratories they use. As noted previously, due to the
ACID-BASE STATE
importance of homeostasis, normal gap versus increased gap
The first step in the assessment of an acid-base imbalance acidosis indicate vastly different clinical pictures.
for a patient is to determine a primary respiratory versus
primary metabolic etiology. A detailed history and compre- Hyperchloremic (Non-anion Gap) Metabolic Acidosis
hensive physical examination can offer clues to the present- Chloride (Cl) has an important relationship to HCO 3
ing cause. Such evaluation may reveal a neurologic (head because there are few causes of non-anion gap acidosis.
injury, seizures), respiratory (pneumonia, congenital mal- In acidotic states with low HCO 3 values, a non-anion
formation), cardiovascular (septic shock, myocarditis), gas- gap acidosis state (or a normal anion gap) can exist through
trointestinal (diarrhea, ingestion), or renal cause (chronic the buffering effect of Cl and is termed hyperchloremic non-
kidney injury, underlying renal disease) of the imbalance. anion gap acidosis. In these conditions, the relative ratio of the
For example, respiratory compromise from a neurologic anions and the cations remains intact, with the anions bal-
cause such as seizures or traumatic brain injury leading to anced by an increase of Cl equal to the decrease in HCO 3 .
reduced consciousness may result in the body’s inability to Non-anion gap acidosis can result from decreased HCO 3
ventilate properly. This prompts a primary respiratory aci- either through gastrointestinal or renal losses. Both of these
dosis. In comparison, gastrointestinal causes can result in types of losses result in a buffering with Cl ions. Gastro-
primary metabolic alkalosis such as through loss of hydro- intestinal (postpyloric) losses include diarrhea and conditions
chloric acid (HCl) by vomiting. such as short gut syndrome. Renal losses include chronic
renal insufficiency and renal tubular acidosis.
A common type of hyperchloremic non-anion gap aci-
dosis occurs in the hospitalized patient who receives infu-
TABLE 2. Generalized Primary and sions of large amounts of normal saline. In these cases, the
Compensatory Relationships Between relative Cl load from the exogenous fluid results in a
HCO3 and PCO2
decrease in HCO 3 to maintain plasma electroneutrality.
Often this phenomenon occurs when large amounts of
PCO2 HCO
3 normal saline are delivered to a patient who has metabolic
Respiratory Acidosis [ (primary) [ (compensatory) acidosis attributed to dehydration or starvation (presumed
Respiratory Alkalosis Y (primary) Y (compensatory)
due to lactic acidosis from hypoperfusion or ketoacidosis
from starvation). These initial metabolic acidosis states
Metabolic Acidosis Y (compensatory) Y (primary)
generally are high-anion gap acidoses, with an imbalance
Metabolic Alkalosis [ (compensatory) [ (primary) in the plasma electroneutrality state due to the contribution
HCO of lactic acids and ketoacids. Following a large infusion of
3 ¼bicarbonate, PCO2¼partial pressure of carbon dioxide
normal saline, the anion gap resolves, but there may be a

Vol. 37 No. 9 SEPTEMBER 2016 363


decrease in HCO 3 values due to the addition of large amounts and creation of ketone bodies. Complicating the diabetic ke-
of Cl. Thus, clinicians who do not take into account the anion toacidosis state is the presence of hyperglycemia (from an
gap when assessing the acid-base status after fluid resuscita- inability to transport glucose intracellularly), which leads to high
tion can be deceived into thinking that the acidosis state has serum osmolality. The resultant osmotic diuresis can progress
worsened by examining only the HCO 3 value. Instead, the to hypovolemic shock and lactic acidosis from anaerobic metab-
presence of a normal anion gap (especially if the patient olism due to hypoperfusion.
previously exhibited gap acidosis) following hydration with Less commonly, underexcretion of acid leading to gap
normal saline indicates an improving clinical condition. acidosis can occur from renal failure or impaired clearance
Although the evidence is unclear regarding whether hyper- of acid (lactate) through the liver due to liver failure. Other
chloremic non-anion gap acidosis from saline resuscitation causes of gap acidosis include inborn errors of metabolism,
worsens outcome, clinicians who understand the physiology and ingestion of toxins, such as those represented in the
may avoid this sequela as much as possible to limit possible mnemonic MUDPILES, leading to acid formation.
adverse effects (eg, increased respiratory work or increased
intracranial pressure). Respiratory Compensation
When exposed to a low pH from metabolic acidosis, the body
Elevated Anion Gap Acidosis responds with an alkalotic respiratory state through hyper-
The presence of an increased gap acidosis (anion gap higher ventilation. This response can occur over minutes to hours,
than normal) is due to an increase of unmeasured anions. depending on an intact neurologic response and the ability of
This can occur following an increase in acid production or an the body to maintain hyperventilation. Through hyperventi-
underexcretion of acid. The classic mnemonic for causes lation, the body reduces the PCO2 within the system, thereby
of gap acidosis is MUDPILES (methanol, uremia, diabetic creating respiratory alkalosis. The respiratory alkalosis com-
ketoacidosis, paraldehyde, isoniazid, lactic acidosis, ethylene bined with the primary metabolic acidosis works to return the
glycol, salicylates). Although most causes on this list repre- body’s pH back to (but never past) normal.
sent exogenous ingestion of acids, physiologically high-anion The classic example of a respiratory compensation to
gap acidosis due to increased production of endogenous acids metabolic acidosis is the diabetic ketoacidosis state. Affec-
is more common. The 2 primary sources for such endoge- ted patients often present with deep hyperventilation, called
nous production are lactic acidosis and ketoacidosis. Kussmaul breathing. The deep and fast breathing pattern
Lactic acidosis can be due to a hypoxia event or shock serves to increase the minute ventilation by increasing both
state, among other causes. Cells produce lactic acid when the volume of each breath (tidal volume) and the number of
energy creation shifts from aerobic to anaerobic metabo- breaths (rate) in a minute. In these cases, neurologic injury can
lism. Although this most commonly occurs when tissue limit the compensatory mechanism for the metabolic derange-
oxygen delivery falls to a critical level, anaerobic metabo- ment. Specifically, with severe diabetic ketoacidosis, cerebral
lism and increased lactic acid production can also occur in edema is one sequela that may result in an inability to direct
hypermetabolic states. Lactate can be produced from cells an appropriate respiratory compensation. When neurologic
in the brain, skeletal muscle, and gastrointestinal tissue injury occurs, the patient may have severe acidosis without an
exposed to hypoxia. The cause of the hypoxic state can be appropriately low PCO2 as the respiratory system attempts to
multifactorial. For example, direct hypoxia such as drown- buffer the metabolic acidosis.
ing or respiratory failure may result in lactic acidosis. In
addition, the failure of mitochondrial function, as occurs in
METABOLIC ALKALOSIS
cyanide poisoning, can lead to anaerobic metabolism and
the subsequent creation of lactic acid. Salicylate toxicity Metabolic alkalosis, as revealed by the Henderson-Hasselbalch
similarly causes failure of oxidative phosphorylation that equation, is due to an increase in the HCO 3 content of the
creates subsequent lactic acidosis. serum, resulting in an increase in pH. The causes include
Ketoacidosis occurs in diabetic ketoacidosis or after a gastrointestinal or renal losses of Cl. Gastrointestinal losses
prolonged starvation state. In those who have diabetes, the can occur through profuse amounts of emesis or Cl-losing
inability to produce insulin results in a state of cellular diarrhea. Renal tubular losses can occur through the use of
starvation due to the body’s failure to use glucose. Conse- diuretics. The subsequent compensation to the Cl loss de-
quently, the body resorts to the breakdown of lipids, produc- signed to maintain electroneutrality is to increase the amount
ing ketoacids. True starvation states in which a glucose source of HCO 3 buffer, leading to a higher pH. Similarly, a net gain
is limited also move the body toward breakdown of lipids of cations can also result in metabolic alkalosis. Causes

364 Pediatrics in Review


include large amounts of lactated Ringer solution and the By hypoventilating, the body drives up the PCO2, thereby
milk-alkali syndrome. creating respiratory acidosis. Combined with the primary
A commonly reviewed cause of metabolic alkalosis in the metabolic alkalosis, the respiratory acidosis works to return
pediatric setting is hypochloremic alkalosis from pyloric steno- the body’s pH back to normal.
sis. In normal physiology, HCl is excreted from the gastric
lumen and neutralized by HCO 3 secreted by the pancreas. In
RESPIRATORY ACIDOSIS
pyloric stenosis, there is a loss of HCl through vomiting and
concurrent decrease in secretion of HCO 3 , resulting in a Respiratory acidosis is the result of an increase in PCO2
decrease in serum Cl and an increase in serum HCO 3 . This leading to a lower pH. This accumulation can be related to
combination results in the classic hypochloremic alkalosis of increased production or decreased elimination of CO2
pyloric stenosis. Preoperative hydration to correct both the through the respiratory system.
dehydration and hypochloremic alkalosis is important for pre- Increased production of CO2 can be due to multiple
vention of postoperative apnea because the respiratory compen- causes. The predominant cause within the pediatric pop-
sation for metabolic alkalosis is hypoventilation (to raise PCO2). ulation is increasing cellular metabolic activity, which can
Another common cause of alkalosis, especially in the be seen in infection or fever. Increased production of CO2
pediatric critical care setting, is the contraction alkalosis can also be related to the carbohydrate load of the body. This
induced by diuretic use. Contraction alkalosis occurs when derangement is often due to iatrogenic causes such as
there is loss of fluids that contain a proportionately lower parenteral nutrition in which an excess of carbohydrate
amount of HCO 3 than the serum concentration. With rela- relative to the body’s need is given. With the body’s ability
tively more free fluid lost, the concentration of HCO 3 re- to eliminate CO2 and increase minute ventilation to account
maining in the serum increases. Furthermore, the interplay of for increased production, an overproduction of CO2 by itself
the renin-angiotensin system, as triggered by the hypovolemic rarely leads to respiratory acidosis unless the body’s com-
state sensed by the body, further increases absorption of pensation is limited (such as in a paralyzed patient). Instead,
HCO 3 through the process of hydrogen ion excretion. Thus, respiratory acidosis more often results from an inability to
the combination of these factors created by diuretic use results remove CO2 from the bloodstream.
in a higher-than-normal HCO 3 value and metabolic alkalosis. Decreased elimination of CO2 can have several causes.
Although increased morbidity and mortality are associ- Elimination of CO2 is based on 3 components of the body
ated with the presence of metabolic alkalosis, most of these working in synchrony: the neurologic component (in rec-
events are self-correcting without clinically significant se- ognizing the need to eliminate CO2), the musculoskeletal
quelae. One key exception to this rule is from pyloric ste- component (in physically moving the chest to create appro-
nosis. Within the critical care setting, metabolic alkalosis priate minute ventilation), and the alveolar component (in
can also be brought on by the use of diuretics before ex- allowing for diffusion of CO2 out of the bloodstream).
tubation in an attempt to improve respiratory mechanics Failure of any of these components can result in respiratory
and pulmonary function in a patient who has had previous acidosis. Neurologic causes include injuries (eg, traumatic
fluid overload. In these instances, acetazolamide is often or stroke), seizures, narcotics, and other pharmacologic
used to waste HCO 3 through the renal system by inhibiting agents causing neurologic depression. Musculoskeletal fail-
the carbonic anhydrase enzyme responsible for proximal ure can result from acute causes such as flail chest or chest
tubule absorption of HCO 3 . The rationale for this approach wall edema or underlying musculoskeletal disorders such as
is to prevent acute hypercapnic respiratory failure follow- myasthenia gravis or muscular dystrophy. However, most
ing extubation. Because the respiratory compensation for commonly, respiratory acidosis occurs from the decreased
metabolic alkalosis is to hypoventilate (and raise PCO2), this ability to eliminate CO2 through the alveoli, such as asso-
acute rise in PCO2 can lead to hypercapnic respiratory failure. ciated with pneumonia, pulmonary edema, and acute respi-
Thus, acetazolamide is commonly used before extubation ratory distress syndrome.
if significant contraction alkalosis from diuretics exists. Treatment for respiratory acidosis involves enhancing
CO2 elimination and treating the underlying cause. For
Respiratory Compensation causes such as pneumonia, treatment with appropriate
Respiratory acidosis through hypoventilation occurs in antibiotics in addition to the short-term use of positive
response to metabolic alkalosis. In response to a high pH, the pressure or mechanical ventilation can improve CO2 elim-
body’s attempt at equilibrium mandates a move toward ination. However, there are instances when, despite maximal
an acidotic respiratory state through hypoventilation. mechanical ventilation, CO2 elimination cannot balance CO2

Vol. 37 No. 9 SEPTEMBER 2016 365


production. Although not frequent, these instances may re- injuries (including meningitis or some traumatic brain injury
quire permissive hypercapnia (high PCO2 with low pH) to or tumors) can cause hyperventilation, depending on the
limit further pulmonary injury from aggressive mechanical location and severity of the insult. Mild respiratory alkalosis
ventilation. Similarly, permissive hypercapnia is a recom- frequently occurs in disease with increased lung water (eg,
mended approach to a neonatal and pediatric ventilation pneumonia, pulmonary edema) due to alveolar stretch recep-
strategy to limit injury from mechanical ventilation. Studies tors enhancing the respiratory drive.
have shown that permissive hypercapnia is well tolerated,
with limited morbidity and mortality in an otherwise healthy Metabolic Compensation
patient (without underlying neurologic pathology or pulmo- In response to a high pH from a primary respiratory alkalosis,
nary hypertension), as long as hypoxia is avoided and hemo- the body’s attempt to equilibrate mandates a move toward an
dynamics are normal. acidotic metabolic state through elimination of HCO 3 . The
primary respiratory alkalosis combined with the metabolic
Metabolic Compensation acidosis works to return the body’s pH to normal. Again, such
In response to a low pH from primary respiratory acidosis, metabolic compensation occurs over days compared to the
the body attempts to achieve equilibrium by moving toward minutes to hours needed in respiratory compensation.
an alkalotic metabolic state through retention of HCO 3
within the metabolic system. By retaining HCO 3 through
CONCLUSION
the renal system, the body can drive up the HCO 3 , thereby
creating metabolic alkalosis based on the Henderson- Understanding the acid-base status of a patient is important
Hasselbalch equation. The primary respiratory acidosis both for those who are critically ill and those who have
combined with the metabolic alkalosis works to return the general metabolic derangements. Although seemingly com-
body’s pH back to normal. plex, an understanding of 2 principles, the Henderson-
Of note, compensation of the metabolic system to a Hasselbalch equation and the anion gap calculation, can
primary respiratory acid-base disorder is much slower than offer clinicians a view into possible causes of the acid-base
compensation of the respiratory system to a primary met- imbalance. Just as important is a firm grasp of the interde-
abolic acid-base disorder. Therefore, in acute respiratory pendence of the metabolic and respiratory systems.
acidosis, the pH is often well below the normal range.
In chronic respiratory disease, such as chronic lung
disease, the clinician may notice a chronically compensated
state. In these cases, the patient maintains a pH within
Summary
normal ranges but has a persistently elevated PCO2 due to the
• A fundamental understanding of acid-base balance is vital to
underlying chronic condition. Over time, the body main- caring for pediatric patients with critical illnesses as well as for
tains homeostasis and the need for a normal pH by chron- managing electrolyte disturbances in those who are not critically
ically retaining HCO 3 . On blood gas examination, such ill. (1)(2)
patients often have a PCO2 in the range of 50 to 60 mm Hg • On the basis of strong research evidence, expert opinion, and
with HCO 3 in the range of 30 to 40 mEq/L (30–40 mmol/L)
consensus, acidosis occurs when the pH value is lower than
at baseline. normal when examining blood gases. In contrast, alkalosis occurs
when the pH value is higher than normal. It is important to
characterize pH as a combination of partial pressure of carbon
RESPIRATORY ALKALOSIS dioxide (PCO2) and bicarbonate (HCO3 ). (1)(2)

• On the basis of strong research evidence, expert opinion, and


Respiratory alkalosis is the result of an excessive elimination consensus, the Henderson-Hasselbalch equation by itself is
of PCO2 leading to a higher pH (“hyperventilation”). This insufficient to completely describe a patient’s metabolic acid-
inappropriate elimination is much less common than any base state. The anion gap further describes the interactions of the
other acid-base derangements. Causes for primary respiratory measured positive charges (cations) and negative charges
(anions) to the unmeasured charged particles. (1)(2)
alkalosis typically involve an increased respiratory drive due to
• On the basis of expert opinion and consensus, the first step in
toxins or primary central nervous system events. Salicylate
the assessment of an acid-base imbalance is to determine a
intoxication and hyperammonemia can promote hyperventi-
primary respiratory versus primary metabolic cause. A detailed
lation. Anxiety and stress can also lead to hyperventilation and history and comprehensive physical examination can offer clues
are typical causes related to the distress a blood draw can to suggest the presenting cause. (1)(2)
induce in a pediatric patient. In addition, some neurologic

366 Pediatrics in Review


• On the basis of expert opinion and consensus, understanding of • On the basis of strong research evidence, expert opinion,
metabolic acidosis requires the use of the anion gap in addition and consensus, respiratory acidosis is the result of an increase
to knowing the pH state. (1)(2) in PCO2 leading to a lower pH. This accumulation can be due to
increased production (such as a higher metabolic state) or
 A common type of hyperchloremic non-anion gap acidosis occurs
in the hospitalized patient with infusion of large amounts of decreased elimination of CO2 through the respiratory system.
normal saline. In these cases, the relative chloride load from the Compensation for primary respiratory acidosis comes from the
exogenous fluid results in a decrease in HCO metabolic system. In response to a low pH, the body’s
3 to maintain plasma
electroneutrality. attempt to reach equilibrium mandates a move toward
an alkalotic metabolic state through retention of HCO 3 .
 The presence of an increased gap metabolic acidosis (anion gap Compensation of the metabolic system to a
higher than normal) is due to an increase of unmeasured anions.
primary respiratory acid-base disorder is much slower
This can occur following an increase in acid production or
than the compensation of the respiratory system to a
underexcretion of acid. The classic mnemonic for causes of gap
primary metabolic acid-base disorder. Therefore, in acute
acidosis is MUDPILES (methanol, uremia, diabetic ketoacidosis,
respiratory acidosis, the pH is often well below the normal
paraldehyde, isoniazid, lactic acidosis, ethylene glycol,
range. (1)(2)
salicylates). Although most causes on this list represent
exogenous ingestion of acids, physiologically high-anion • On the basis of strong research evidence, expert opinion,
gap acidosis due to increased production of endogenous acids is and consensus, respiratory alkalosis is the result of
more common. The 2 primary sources of such endogenous an excessive elimination of PCO2 leading to a higher pH
production are lactic acidosis and ketoacidosis. (“hyperventilation”). This inappropriate elimination is much
less common but can occur through an increased
 In response to a low pH from metabolic acidosis, the body’s attempt respiratory drive due to toxins or primary central nervous
to reach equilibrium mandates a move toward an alkalotic
system events. Compensation for a primary respiratory
respiratory state through hyperventilation. This response can occur
alkalosis comes from the metabolic system. In response to a
over minutes to hours, depending on an intact neurologic response
high pH, the body’s attempt to equilibrate mandates a move
and the ability of the body to maintain hyperventilation.
toward an acidotic metabolic state through elimination of
• On the basis of strong research evidence, expert opinion, and HCO 3 . (1)(2)
consensus, metabolic alkalosis, as revealed by the Henderson-
Hasselbalch equation, points to an increase in the HCO3 content of
the serum, resulting in an increase in pH. Two common causes
include hypochloremic alkalosis from pyloric stenosis and
contraction alkalosis induced by diuretics use. Compensation for the
metabolic alkalosis comes from the respiratory system. In response
to a high pH, the body’s attempt to equilibrate mandates a move References for this article are at http://pedsinreview.aappublications.
toward an acidotic respiratory state through hypoventilation. (1)(2) org/content/37/9/361.

Vol. 37 No. 9 SEPTEMBER 2016 367


PIR Quiz
There are two ways to access the journal CME quizzes:
1. Individual CME quizzes are available via a handy blue CME link under the article title in the Table of Contents of any issue.
2. To access all CME articles, click “Journal CME” from Gateway’s orange main menu or go directly to: http://www.aappublications.
org/content/journal-cme.

1. A 6-year-old girl is evaluated in the emergency department after 2 days of vomiting and REQUIREMENTS: Learners
diarrhea. She has had low oral intake and scant urine output for the past 16 hours. An can take Pediatrics in
electrolyte panel is ordered. The results are as follows: Review quizzes and claim
Sodium 135 mEq/L (135 mmol/L) credit online only at:
Potassium 4.8 mEq/L (4.8 mmol/L) http://pedsinreview.org.
Chloride 110 mEq/L (110 mmol/L)
Bicarbonate 8 mEq/L (8 mmol/L)
To successfully complete
Blood urea nitrogen 29 mg/dL (10.4 mmol/L)
2016 Pediatrics in Review
Creatinine 1.0 mg/dL (88.4 mmol/L)
articles for AMA PRA
Calcium 8.5 mg/dL (2.1 mmol/L)
Category 1 CreditTM,
Albumin 4.5 g/dL (45 g/L)
learners must
The anion gap in this patient equals:
demonstrate a minimum
A. 21.8 mEq/L (21.8 mmol/L). performance level of 60%
B. 25 mEq/L (25 mmol/L). or higher on this
C. 25.5 mEq/L (25.5 mmol/L). assessment, which
D. 26.3 mEq/L (26.3 mmol/L). measures achievement of
E. 30.3 mEq/L (30.3 mmol/L). the educational purpose
2. A 4-year-old boy is admitted to the pediatric intensive care unit with overwhelming sepsis and/or objectives of this
due to Streptococcus pneumoniae. The anion gap is normal despite a definite metabolic activity. If you score less
acidosis. Your colleague states that this is likely a falsely low anion gap. Which of the than 60% on the
following is the most likely explanation for the low anion gap? assessment, you will be
A. Conservation of bicarbonate due to decreased urine output. given additional
B. Decrease in unmeasured anions due to hypoalbuminemia. opportunities to answer
C. Increase in carbon dioxide due to respiratory depression. questions until an overall
D. Increase in lactate concentration due to decreased perfusion of vital organs. 60% or greater score is
E. Increase in serum protein concentrations due to inflammation. achieved.
3. A previously healthy 3-year-old girl is admitted to the hospital for dehydration. Her initial
bicarbonate measures 9 mEq/L (9 mmol/L) with an anion gap of 24 mEq/L (24 mmol/L). This journal-based CME
After vigorous resuscitation with 2 bolus infusions of 20 mL/kg normal saline, her activity is available
bicarbonate is 7 mEq/L (7 mmol/L) and the anion gap is 16 mEq/L (16 mmol/L). What is the through Dec. 31, 2018,
most likely explanation for her second set of laboratory results? however, credit will be
A. An increase in her circulatory volume has diluted her chemistry values. recorded in the year in
B. An increased lactate concentration has displaced bicarbonate. which the learner
C. She is becoming more acidotic due to deterioration in her overall status. completes the quiz.
D. She is continuing to lose bicarbonate due to persistent diarrhea.
E. The relative chloride load caused a decrease in bicarbonate to maintain
electroneutrality.
4. An 18-month-old male toddler is brought to the emergency department by his
grandparents, who found him in their bathroom after he opened a bottle of “muscle pain
relief” wintergreen oil (containing methyl salicylate) and ingested some of its contents. The
boy is sleepy and irritable when aroused. Laboratory studies reveal a metabolic acidosis
with an elevated anion gap. Keeping in mind the “MUDPILES” mnemonic for causes of an
elevated anion gap acidosis, which of the following causes for this child’s metabolic
condition is most likely?
A. Ethylene glycol.
B. Ketoacidosis.
C. Lactic acidosis.
D. Methanol.
E. Uremia.

368 Pediatrics in Review


5. A 6-week-old male infant presents with vomiting that began insidiously but progressed to
projectile vomiting of all feedings. Ultrasonography confirms the diagnosis of hypertrophic
pyloric stenosis. Which of the following is the expected primary finding in this disorder?
A. Hyperchloremic metabolic acidosis.
B. Hypochloremic metabolic alkalosis.
C. Hypercapneic respiratory acidosis.
D. Hypocapneic respiratory alkalosis.
E. Metabolic contraction alkalosis.

Vol. 37 No. 9 SEPTEMBER 2016 369


Evaluation and Initial Management of
Hypopituitarism
Melinda Pierce, MD,* Lisa Madison, MD*
*OHSU Department of Pediatrics, Division of Pediatric Endocrinology, Portland, OR.

Education Gap
Signs and symptoms of hypopituitarism, in particular diabetes insipidus
and decreased linear growth velocity, are frequently overlooked in
patients with craniopharyngioma. (1)

Objectives After completing this article, readers should be able to:

1. Describe the various pathophysiologic causes associated with


hypopituitarism, including craniopharyngioma.
2. Recognize the clinical features of pituitary hormone deficiencies.
3. Describe the principles of hormone replacement therapy for
hypopituitarism.

INTRODUCTION

Hypopituitarism affects between 1 in 4,000 and 1 in 10,000 live births, with


increasing incidence with age. (2) The pituitary, sometimes referred to as the
master gland, is positioned in the anterior midline of the brain in the sella turcica.
The pituitary controls endocrine function through the release of multiple hor-
mones that can have a direct growth effect on target tissue (trophic effects) or
stimulate that tissue to release a hormone that has its effects on other tissues
(tropic effects). The anterior pituitary forms as an outpouching of the embryonic
oral cavity (Rathke pouch). It is responsible for the release of growth hormone
(GH), corticotropin, thyrotropin, prolactin, and the gonadotropins luteinizing
hormone (LH) and follicle-stimulating hormone (FSH). The posterior pituitary
is composed of neuronal projections from the hypothalamus and is responsible
for the storage and secretion of oxytocin and antidiuretic hormone (ADH). This
article examines the pathologic causes of pituitary dysfunction, their diagnosis,
and treatment.

CAUSES OF HYPOPITUITARISM
AUTHOR DISCLOSURE Drs Pierce and
Congenital Madison disclosed no financial relationships
relevant to this article. This commentary does
Congenital hypopituitarism most often results from genetic or embryologic
not contain a discussion of an unapproved/
pathologies. Septo-optic dysplasia (SOD) is the most common congenital cause investigative use of a commercial product/
of hypopituitarism, with an incidence as high as 1 in 10,000. SOD results in device.

370 Pediatrics in Review


variable hormonal deficiencies, absence of the septum pellu- surgical manipulation as well as motor vehicle collisions, child
cidum (Figure), and hypoplasia of the optic nerves, often abuse, and sports-related injuries. There are cases of idiopathic
resulting in blindness. (3) Although SOD can be associated pituitary hormone deficiencies, most commonly isolated idio-
with genetic mutations (in HESX1, OTX2, and SOX2), it is most pathic GH deficiency, but this is always a diagnosis of exclusion.
often multifactorial and is associated with abnormal midline Diabetes insipidus (DI) can occur from any injury to the
brain development. (4) Other defects in midline formation pituitary stalk that damages neuronal projections respon-
(such as midline cleft lip and/or palate, vascular malforma- sible for the delivery of ADH to the storage vesicles in the
tions, absence of the corpus callosum) can also be associated posterior pituitary. In cases of isolated DI, an identified
with an embryologic malformation of the pituitary gland. cause is found at the time of diagnosis in 50% to 90% of
In cases of SOD or facial midline defects, further screen- cases, with the most common causes being anatomic abnor-
ing should be undertaken that includes testing for pitui- malities, central brain tumors, and infiltrative diseases. (7)(8)
tary hormone deficiencies (see the section on “Evaluation”), Any mass that comes in contact with the hypothalamus
ophthalmologic evaluation, and imaging of the central ner- or pituitary can place pressure on the sensitive endocrine
vous system (CNS) (eg, magnetic resonance imaging [MRI]). cells, leading to dysfunction. GH-producing cells are the
Such screening should occur in the neonatal period to prevent most sensitive to damage from any cause. Common pediatric
complications from untreated hormone deficiency and to tumors include craniopharyngioma, primary brain tumors
assess possible vision deficits that may require accommo- such as germinomas and optic gliomas, functional and non-
dation. Rare genetic causes of hypopituitarism include dele- functional pituitary macroadenomas, and cerebral metastases
tions of the “POU domain class 1 transcription factor,” PIT-1 of other primary cancers such as lymphoma. Prolactinomas
or POU1F1, and the “homeobox protein prophet of PIT-1,” deserve special mention because they are often not of a
encoded by the PROP1 gene. (5)(6) GH, prolactin, and size to cause problems from mass effect. However, hyperpro-
TSH deficiencies characterize PIT-1/POU1F1 deletion, lactinemia can cause negative feedback and inhibition of the
whereas PROP1 deletions can manifest as any combination hypothalamic-pituitary-gonadal axis, resulting in hypogonado-
of deficiencies of the anterior pituitary hormones. tropic hypogonadism with delayed or arrested puberty.
Hyperprolactinemia is also associated with galactorrhea.
Acquired Craniopharyngioma is the most common tumor asso-
Later in life, anterior pituitary hormone deficiency can arise ciated with multiple pituitary deficiencies in children. A
from any cause that damages the pituitary, including mass effect craniopharyngioma arises from the same oral ectoderm as the
(from tumor), infection, autoimmune disease, infiltrative dis- anterior pituitary. In a case series of 42 children with cranio-
ease, chemotherapy and radiation exposure, and trauma. Any pharyngioma, only 10% sought medical treatment for symp-
trauma or damage to the sella, pituitary, pituitary blood supply, toms, although more than 50% had evidence of growth failure
or hypothalamus can result in hypopituitarism. This includes before identification. (9) GH deficiency is the most common
pituitary deficiency seen in craniopharyngioma, affecting up
to 75% of patients. Other pituitary hormone deficiencies
include gonadotropin deficiency, thyrotropin deficiency, cor-
ticotropin deficiency, and DI, with the incidence of each
increasing from baseline after surgical intervention. (9)
Because surgical excision of the mass can disturb the pituitary
and the stalk, the patient should be completely evaluated for
deficiencies both before and after surgery to identify any
new deficiencies. This is particularly true of GH and ADH.
Even when a brain tumor does not itself damage the
pituitary, tumor treatment may result in hypopituitarism.
Any radiation exposure to the pituitary can cause permanent
pituitary damage, with GH-producing cells again being
most sensitive. Although 100% of children who are exposed
to more than 30 Gy eventually develop GH deficiency, any
brain exposure greater than 22 Gy should prompt evalu-
ation for hormonal deficits. (10) Common chemothera-
Figure. Magnetic resonance imaging of an infant at 4 days of age
showing absence of the septum pellucidum. peutic drugs are not currently believed to result in central

Vol. 37 No. 9 SEPTEMBER 2016 371


deficiency, but multiple chemotherapy modalities can dam- inappropriate ADH secretion (SIADH). Urine output may
age the target organs. High-dose corticosteroids for induction be normal or decreased in volume but is concentrated (urine
chemotherapy in acute lymphoblastic leukemia can suppress osmolality greater than serum) and often accompanied by
the hypothalamic-pituitary-adrenal axis, but this treatment is hyponatremia. After the death of the ADH neurons, the third
not associated with other pituitary hormone deficiencies. phase is characterized by permanent DI.
Certain infiltrative diseases also have a predilection for the
pituitary, particularly Langerhans cell histiocytosis (LCH),
EVALUATION
which results from overproliferation of Langerhans cells.
The classic triad of LCH symptoms is also referred to as Hormones
multifocal unisystem disease and includes DI, exophthalmos, Each individual hormone should be assessed during labo-
and lytic bone lesions. GH deficiency is also common in ratory testing. Although idiopathic isolated GH deficiency
LCH. In cases that were evaluated retrospectively after an initial is seen rather commonly, other isolated pituitary hormone
diagnosis of idiopathic DI, approximately one-third of patients deficiencies are relatively rare. Accordingly, clinicians should
were subsequently found to have a pathologic etiology, with test for all anterior hormone deficiencies once any single
approximately 50% of those found to have histiocytosis. (8) deficiency, including GH, is identified. It is also critical that
CNS infection, such as meningitis, is classically associated testing for corticotropin, LH, FSH, thyrotropin, and GH
with the development of DI, but there is a risk of damage to include evaluation of both the pituitary hormone itself and
other hormone-producing cell types as well. Infiltrative infec- the target organ hormones. Frankly low concentrations of
tions such as tuberculosis can lead to pituitary destruction. pituitary hormones are not necessary to diagnose deficiency;
Hypophysitis is a rare cause of autoimmune damage to they may be inappropriately normal in the setting of target
the pituitary, similar to other autoimmune lymphocytic hormone deficiency.
processes such as Hashimoto thyroiditis or Addison dis- GH has a typically pulsatile release that makes evaluation of
ease. In most cases, hypophysitis is a diagnosis of exclusion a random value difficult to interpret. Instead, screening for
when symptoms of hypopituitarism are present with CNS GH deficiency relies on evaluation of the secondary growth
symptoms such as headache, nausea, or vision problems. factors insulin-like growth factor-1 (IGF-1) and insulin-like
No reliable antibody assays are available to test for hypo- growth factor-binding protein-3 (IGFBP-3). Both are produced
physitis in peripheral blood and fewer than 400 cases have by the liver in response to the presence of GH. These growth
been reported in the literature. (11) factors have longer half-lives than GH and are maintained at a
steady level in response to normal pulsatile GH secretion. If
growth factor values are low, formal testing for GH can be
SYMPTOMS OF HYPOPITUITARISM
performed through stimulation of GH secretion. This is
Symptoms of hypopituitarism are related to both hormonal typically achieved by a combination of medications that
deficits and the timing of the insult to the pituitary. In older either directly promote GH secretion or create stress in
children, when not associated with a known disorder, hypo- the body (such as hypoglycemia) that results in GH secretion.
pituitarism is often identified after evaluation for poor linear Cortisol has circadian release that increases the difficulty of
growth. The symptoms most commonly seen in each in- assessment with a random value, similar to GH. However, the
dividual hormone deficiency are listed in Table 1. Other tropic hormone corticotropin is required by the adrenal glands
symptoms include those from the underlying pathologic to maintain secretory function. If corticotropin is missing,
cause of the hypopituitarism, such as visual deficits or head- adrenals atrophy and cortisol release is impaired. Corticotro-
aches from a midline mass, fever or sepsis associated with pin deficiency can result in adrenal crisis if profound or
meningitis, or bony lesions from LCH. prolonged. Adrenal crisis is characterized by hypotension
Surgical manipulation of the posterior pituitary creates a and cardiac instability. Laboratory testing is via provocative
classic triphasic ADH response. The first phase, manifested testing of the adrenal glands through administration of
as DI, occurs within 24 hours of surgery. Patients have exogenous corticotropin. If central deficiency (corticotropin
excessive urine output, often more than 5 to 10 mL/kg per deficiency) is suspected, a small dose of corticotropin is used
hour. The urine is dilute (specific gravity <1.010 and urine (1 mg), based on the concept that a normally functioning gland
osmolality <100 mOsm/kg), with hypernatremia and in- is sensitive but an atrophic gland is not. (12) This is in contrast
creased serum osmolality (often >300 mOsm/kg). The sec- to the 250-mg corticotropin stimulation test that is performed
ond phase is associated with release of preformed ADH from if there is suspicion of primary adrenal insufficiency. With
the dying neurons, resulting in a transient syndrome of either dose, the cortisol is measured at 30 minutes, with a

372 Pediatrics in Review


TABLE 1. Common Signs and Symptoms Associated with Hormone Deficiencies
HORMONE MISSING SIGNS AND SYMPTOMS

Growth hormone Linear growth failure, increased adiposity, decreased muscle mass, fatigue
If congenital: neonatal hypoglycemia, micropenis
Corticotropin Nausea, vomiting, weight loss, prolonged duration of common illnesses,
hypotension, fatigue
Thyrotropin Fatigue, dry hair, dry skin, linear growth failure, constipation, weight gain
despite decreased intake, bradycardia
If congenital and untreated: development delays, intellectual disability
Prolactin Inability to lactate after pregnancy
Luteinizing hormone/Follicle-stimulating hormone Delayed or absent pubertal development with lack of pubertal growth spurt,
secondary amenorrhea, decreased libido, osteoporosis
If congenital: micropenis, undescended testicles
Antidiuretic hormone Polyuria, polydipsia, nocturia, dehydration, hypernatremia

passing value of 18 mg/dL (496.6 nmol/L). (13) In addition, osmolality. Urine is inappropriately dilute for the serum
measurement of both baseline corticotropin and cortisol sodium concentration. The water deprivation test involves
concentrations can aid in distinguishing between primary monitoring of a patient during restricted access to free water.
and secondary adrenal insufficiency because corticotropin Urine output, serum sodium, and serum and urine osmolality
will be elevated in the setting of primary disease. Ran- are frequently assessed to determine if the patient develops
dom corticotropin values, especially if measured without a hypernatremia with elevated serum osmolality in the setting
concomitant cortisol value, are rarely helpful diagnostically. of inappropriately high urine output with low urine osmo-
Thyrotropin deficiency is manifested by low circulating lality. Because of the risk of developing significant hyper-
thyroid hormone (free thyroxine [T4]) concentrations in the natremia, this test should only be performed in the hospital
setting of a low or normal thyrotropin values. A normal where frequent monitoring is available.
thyrotropin with a low free T4 is considered inappropriately
normal because intact feedback would result in elevation Identification of Acquired Cause
of the thyrotropin above the normal range. In general, when Because most acquired cases of hypopituitarism are asso-
evaluating suspected hypothyroidism, clinicians should ciated with a treatable disease, identifying and treating the
assess both thyrotropin and free T4 to avoid missing central cause is as important as hormone replacement. Often the
causes of hypothyroidism. first step is imaging of the brain via MRI with and without
Testing for gonadotropin deficiency should always be contrast. This can identify the solid and cystic sellar mass
performed using a highly sensitive assay that can measure characteristic of craniopharyngioma (Figure) as well as
results less than 1 mIU/mL. It should include testing for smaller areas of hypoenhancement associated with pitui-
both LH and FSH as well as the patient-appropriate sex tary adenomas. Hypophysitis is extremely rare but can
steroid (estrogen or testosterone). Similar to thyrotropin appear as an enlarged or diffusely hyperenhanced pituitary
deficiency, normal gonadotropin values in the setting of low gland. Skull fractures and other signs of trauma are often
sex steroid values at the normal age of puberty should raise readily evident on imaging, although they can be missed
concern for pituitary insufficiency. when small. Tumor markers for common brain tumors
Central DI from ADH deficiency is primarily manifested include human chorionic gonadotropin and a-fetoprotein.
by the clinical symptoms of polyuria and polydipsia. Normal
serum sodium concentrations can be maintained unless
TREATMENT
the patient is unable to sustain adequate free water intake,
whether due to developmental stage, absent thirst mecha- Treatment for hypopituitarism is, in principle, very sim-
nism, profound illness, or iatrogenic causes such as nil per os ple: replace the target hormones that are deficient. In practice,
status for surgery. In this setting, the excessive loss of free such replacement can be relatively complicated and requires
water causes hypernatremia with resultant increase in plasma close patient monitoring and appropriate dose adjustments.

Vol. 37 No. 9 SEPTEMBER 2016 373


Adrenal Insufficiency replacement. GH should not be administered in the setting
Once the cause has been identified, treatment should be of active malignancy because of theoretical concerns for
directed toward that pathology. However, before undertaking increasing cancer proliferation. Patients are typically advised
any surgical interventions, it is critical to treat any adrenal to wait 1 year after surgical resection of a craniopharyngioma
insufficiency and administer the appropriate stress-dose level and at least 2 years after successful treatment of other malig-
of cortisol replacement both during and immediately follow- nancies before beginning GH replacement therapy. Some of
ing surgery. If neurosurgical intervention is undertaken, post- the most significant adverse effects of GH treatment are
surgical use of high-dose dexamethasone is appropriate for increased risk for glucose intolerance and hyperinsulinism,
replacement stress dosing, although hydrocortisone should slipped capital femoral epiphysis, pseudotumor cerebri, and
be initiated at the end of the corticosteroid weaning period. pancreatitis. Overall, GH replacement in a deficient child is
Hydrocortisone has a half-life in serum of 6 to 8 hours generally considered to be safe.
and, therefore, ideally should be administered 3 times a day
(morning, mid-afternoon, and evening). The maintenance Gonadotropin Deficiency
replacement hydrocortisone dose is 7 to 9 mg/m2 per day. Gonadotropin deficiency results in low concentrations of
If doses cannot be administered in even allotments, the larger sex-specific sex steroids: testosterone in males and estrogen
dose frequently is provided in the morning to better mimic in females. This results in absent puberty manifested by
the normal surge in cortisol upon waking. Stress dosing also failure to develop breast tissue and menstruation in girls
is necessary in times of illness. Typically, doses are doubled and failure to increase testicular size in boys. Deficiencies in
for fevers and tripled for any vomiting or diarrhea. Patients testosterone/estrogen increase the risk for osteoporosis
(and parents in particular) must be trained in the use of as well as metabolic abnormalities. Replacement is started
emergency hydrocortisone injection (approximately 10 times around the time of mean entrance into puberty for the pa-
maintenance) that can be administered intramuscularly if the tient’s sex (ie, age 10 years for girls, age 11–12 years for boys).
patient has suspected adrenal crisis or is unable to take oral Estrogen is available as either a transcutaneous patch or
medications. pill. Many experts prefer the transcutaneous method be-
cause it bypasses the “first-pass hepatic metabolism” seen
Hypothyroidism with oral medication and may result in better replacement
Thyroid hormone is replaced with levothyroxine, the right- with lower risk. Doses start small and are increased approx-
handed isomer of T4. Levothyroxine is well absorbed and imately every 6 months in an effort to mimic the natural
can be converted to the active hormone, triiodothyronine, in progression of secondary sexual characteristics. Once the
target tissues to allow for tissue-specific control of thyroid uterine lining has received sufficient estrogen stimulation,
signaling. Weight-based higher doses of thyroid hormone are there may be breakthrough vaginal bleeding. At this time,
required in younger children, and appropriate replacement is progesterone is typically added to stabilize the uterine lining
critical for normal neurodevelopment in the first 3 years after and is periodically stopped to allow a regular withdrawal
birth. The schedule in Table 2 is modified from the protocol bleeding or menses.
used in cases of congenital hypothyroidism; doses may be less Testosterone replacement is available for boys in either an
for patients with some pituitary function, although the mon- intramuscular or topical preparation. Initial doses are small
itoring frequency is similar. Patients are monitored through (approximately 1/8th–1/4th the normal adult dose of 200 mg
measurement of free T4 alone, with the goal of maintaining testosterone) and if given intramuscularly, are spaced out
the values in the upper half of the normal range. approximately monthly. Doses are increased every 6 months
in an effort to mimic the natural progression of secondary
Growth Hormone Deficiency sexual characteristics and to allow for continued linear growth
Recombinant GH is available to treat patients with GH before the closure of the growth plates.
deficiency. As a peptide hormone, it is currently only
available as a subcutaneous injection that must be admin- Diabetes Insipidus
istered daily. Typically, initial replacement doses range Central DI is treated with synthetic ADH, most commonly
from 0.025 to 0.05 mg/kg per day, with monitoring in the form of oral desmopressin (DDAVP). For patients
of IGF-1 values and growth velocity. Goal IGF-1 values with an intact thirst mechanism, the goal of therapy is to
should be in the upper half of the normal range. Growth provide sufficient relief from polyuria and polydipsia to al-
velocity should be age-appropriate, although it may low the individual to sleep through the night without
be higher than normal in the first 6 to 12 months of nocturia and to participate in normal daily activities without

374 Pediatrics in Review


autoimmune/autoinflammatory or infiltrative disease, and/
TABLE 2. Levothyroxine Dose Recommendations or infection. Among pediatric patients with hypopituitarism,
some of the most common causes are craniopharyngioma,
DAILY WEIGHT- FREQUENCY OF
AGE BASED DOSE MONITORING SOD, and brain tumors (including treatment). Treatment
involves the physiologic replacement of the end-organ hor-
0-12 months 10-15 mg/kg Every 1-3 months
mones that are deficient, with close monitoring for adequacy
1-5 years 4-6 mg/kg Every 3-6 months of replacement.
6-12 years 3-5 mg/kg Every 6-12 months
>12 years 2-3 mg/kg Every 6-12 months

Summary
excessive disruption. Breakthrough polyuria and polydipsia • On the basis of observational studies, (1)(2)(3)(4)(5)(6)(7)(8)
must be allowed at least once during a 24-hour period hypopituitarism has an increasing prevalence with increasing
because it is during this time that the patient self-regulates age.
to maintain a normal serum sodium concentration. The • On the basis of case reports and observational studies, (3)(4)
starting dose of DDAVP is 0.05 to 0.2 mg once daily, and septo-optic dysplasia is the most common congenital cause of
hypopituitarism.
doses are titrated to effect, often requiring twice-daily and
sometimes three times-daily dosing to achieve adequate • On the basis of epidemiologic studies, (1)(2)(3)(4)(5)(6)(7)(8)(9)
(10)(11) other causes of hypopituitarism include genetic or
control of urine output.
anatomic abnormalities (including midline facial defects), trauma,
For patients with impaired thirst due to hypothalamic mass effect, infection, and autoimmune and infiltrative diseases.
injury or patients unable to regulate their own intake (eg, • On the basis of case reports and expert opinion, single detected
those who have developmental disability or are dependent on pituitary hormone deficiencies are rarely isolated and should
gastrostomy tubes), the usual strategy for DDAVP dosing is to prompt evaluation of the other pituitary hormones. Pituitary
provide maximal antidiuresis in concert with a fluid pre- hormone values may be inappropriately normal in the setting of
scription. Maximal antidiuresis allows for approximately 0.5 end-organ hormone deficiency.

mL/kg per hour of urine output, and the corresponding fluid • On the basis of expert opinion and case reports, (9) clinical
features of hypopituitarism can initially be subtle and vary based
prescription provides replacement of urine output plus insen-
on the hormones involved, although poor linear growth is a
sible losses. Initial titration of such a regimen almost always
prominent finding in many deficiencies and should prompt
requires an inpatient stay with careful monitoring of intake consideration of further testing.
and output as well as frequent serum sodium measurement. • On the basis of expert opinion, diabetes insipidus is characterized
by loss of free water from an inability to concentrate the urine that
results in hypernatremic dehydration if untreated.
CONCLUSION

Hypopituitarism is a rare but complex diagnosis in the


pediatric patient that requires thorough evaluation and
treatment. It can result from a multitude of causes, including References for this article are at http://pedsinreview.aappublications.
mass effect, trauma (mechanical, chemical, or radiological), org/content/37/9/370.

Vol. 37 No. 9 SEPTEMBER 2016 375


PIR Quiz
There are two ways to access the journal CME quizzes:
1. Individual CME quizzes are available via a handy blue CME link under the article title in the Table of Contents of any issue.
2. To access all CME articles, click “Journal CME” from Gateway’s orange main menu or go directly to: http://www.aappublications.
org/content/journal-cme.

1. You are preparing a medical student to see a 16-year-old boy with a history of injury to the REQUIREMENTS: Learners
posterior pituitary as a result of head trauma associated with basilar skull fracture. Of the can take Pediatrics in
following, you explain that this region of the pituitary is responsible for the production of: Review quizzes and claim
A. Antidiuretic hormone (ADH). credit online only at:
B. Corticotropin. http://pedsinreview.org.
C. Gonadotropins (follicle-stimulating hormone [FSH], luteinizing hormone [LH]).
D. Growth hormone (GH). To successfully complete
E. Thyrotropin. 2016 Pediatrics in Review
articles for AMA PRA
2. A term newborn presents with hypoglycemia and poor visual fixation. Magnetic resonance Category 1 CreditTM,
imaging (MRI) of the brain shows absence of the septum pellucidum, and laboratory learners must
evaluation demonstrates deficiency of multiple pituitary hormones. Which of the following demonstrate a minimum
is the most appropriate diagnosis for this newborn’s presentation? performance level of 60%
A. Birth trauma. or higher on this
B. Congenital syphilis. assessment, which
C. Genetic deletion resulting in deficient production of anterior pituitary hormones. measures achievement of
D. Kallmann syndrome. the educational purpose
E. Septo-optic dysplasia. and/or objectives of this
activity. If you score less
3. A 12-year-old girl presents with exophthalmos and a lytic lesion in the skull. She is than 60% on the
diagnosed with Langerhans cell histiocytosis. Which of the following pituitary disorders is assessment, you will be
most commonly associated with this condition? given additional
A. Central adrenal insufficiency (corticotropin insufficiency). opportunities to answer
B. Central hypothyroidism (thyrotropin deficiency). questions until an overall
C. Delayed puberty due to gonadotropin deficiency. 60% or greater score is
D. Diabetes insipidus (ADH deficiency). achieved.
E. Hyperprolactinemia (overproduction of prolactin).
This journal-based CME
4. A 7-year-old male with septo-optic dysplasia is scheduled for bilateral indirect hernia activity is available
repair. Which of the following hormone levels is the most important to assess prior to through Dec. 31, 2018,
surgery? however, credit will be
A. GH. recorded in the year in
B. Insulin-like growth factor-1. which the learner
C. LH. completes the quiz.
D. Cortisol.
E. FSH.

5. A 6-year-old boy presented with growth failure, headaches, personality changes, and
visual field deficits. MRI of the brain revealed a tumor involving the pituitary gland. The
tumor was recently resected. In regards to subsequent GH replacement therapy, the most
appropriate statement is that GH therapy:
A. Is associated with an unacceptable risk of adverse events in childhood.
B. Is monitored by measuring random GH concentrations.
C. Should be deferred until 6 months after surgery.
D. Should begin in the immediate postoperative period.
E. Should not be given in the setting of active malignancy.

376 Pediatrics in Review


Chronic and Recurrent Abdominal Pain
Paul E. Hyman, MD*
*Louisiana State University Health Sciences Center and Children’s Hospital of New Orleans, New Orleans, LA.

Education Gap
The past 20 years have witnessed a transition in clinical understanding of
childhood bellyaches. The recently published third iteration of pediatric
Rome criteria provides updated and accurate criteria for symptom-based
diagnosis of chronic and recurrent abdominal pain. In many cases,
primary care clinicians can make symptom-based diagnoses and initiate
treatment on the first visit.

Objectives After completing this article, readers should be able to:

1. Make symptom-based diagnoses of functional abdominal pain


disorders.
2. List a variety of treatment options for children with functional
abdominal pain.
3. Recognize warning signs that discriminate disease from functional
abdominal pain.
4. Understand how psychosocial factors play a role in disability associated
with functional abdominal pain.

EPIDEMIOLOGY

By definition, chronic or recurrent abdominal pain must occur at least 4 times


each month for at least 2 months. Abdominal pain complaints begin as soon as a
child can provide an accurate pain history, usually around age 7 years but occa-
sionally younger. Before that age, children have difficulty separating emotional
distress from physical pain. The differential diagnosis of child and adolescent
abdominal pain is unrelated to age.
One in 10 children visits a clinician because of chronic or recurrent abdominal
pain. (1) How can a clinician screen quickly for disease? The first consideration is
the duration of each episode. If the pain lasts less than 5 minutes, even if it occurs
many times daily, the pain is unlikely to be worrisome. Abdominal pains lasting
just a few minutes may be abdominal wall muscle cramps or high-amplitude-
propagating colon contractions. High-amplitude-propagating contractions are
AUTHOR DISCLOSURE Dr Hyman has waves of muscle contraction starting in the ascending colon, with pressures
disclosed no financial relationships relevant to
greater than 60 mm Hg, that move colonic contents through the colon to the
this article. This commentary does contain a
discussion of an unapproved/investigative rectum. (2) High-amplitude-propagating colon contractions are normal, but they
use of a commercial product/device. cause an urge to defecate for about 2 minutes several times daily. Children who

Vol. 37 No. 9 SEPTEMBER 2016 377


are sensory-sensitive or children who refuse to defecate may physiologic events such as eating or defecation and likely
experience abdominal pain with normal high-amplitude- reflects central nervous system (CNS) origins for pain. If the
propagating contractions. The second consideration is loca- pain comes and goes, ask how often the episodes occur and
tion. The closer the pain location is to the umbilicus, the less how long they last. The next question is, “Does eating make
likely it is due to disease. The final consideration is the time your pain better, worse, or no different?” Pain that worsens
of the pain complaints. Children with functional (non- with eating suggests that the cause is stretching of the
organic) abdominal pain often complain either upon awak- stomach (dyspepsia) or the gastrocolonic response increas-
ening or when going to sleep, but they seem relatively ing pressures in the colon. Next on the question list is, “Does
symptom-free during daily activities. Waking up and going defecation make your pain better, worse, or no different?” If
to sleep share the characteristic that in those quiet min- defecation relieves the pain, the diagnosis is likely to be
utes children assess their bodies for discomfort. Once irritable bowel syndrome (IBS). This can be followed with,
active in the day, healthy children are less aware of body “Does exercise make your pain better, worse, or no differ-
sensations. ent?” If eating and defecation have no effect but exercise
Most chronic or recurrent abdominal pain is not caused makes the pain worse, the diagnosis may be abdominal wall
by disease. In a study of Norwegian children with abdominal pain, unrelated to gastrointestinal factors. If the pain is
pain, 87% met diagnostic criteria for 1 or more functional unchanged by eating, defecation, exercise, or any other
gastrointestinal disorders (FGIDs) on their first visit (Fig 1). physiologic event, it is a disorder termed “functional
(3) Diagnoses change over time to an organic disease in only abdominal pain not otherwise specified.” A helpful fol-
1% to 2% of those diagnosed with an FGID. Inflammatory low-up question is, “What can you do to make your pain
bowel disease occurs in about 1 in 1,000 children. (4) Celiac better?” This question provides an opening for discussion of
disease is found in 1 in 150 children, but most affected how the patient copes, with possibilities such as medicines,
children are asymptomatic. (5) Thus, the chances are slim foods, rest, and distractions. It also is important to ask about
that the next child who comes to the office with a bellyache what makes the pain worse, with possibilities such as food,
has a disease. exercise, academic struggles, bullying, and family stressors
(eg, psychological or physical abuse in the home or separation
from a loved one by death or relocation).
HISTORY AND PHYSICAL EXAMINATION
The patient’s physical appearance is helpful in assessing
A series of simple questions can provide valuable informa- the degree of distress at the time of examination. Those
tion when taking a history from a patient complaining of suffering from chronic pain may appear to be in no distress
pain. The first question should address whether the pain but when queried, they rate their pain as an 8 or 9 out of 10.
is constant or intermittent. Constant pain is unrelated to Clinicians should never challenge a patient’s pain rating
because chronic pain differs from acute pain in its pheno-
type and challenging the patient’s story can result in a failed
therapeutic alliance. The patient who has an acute abdomen
appears ill, with tachycardia, flexed legs, and a facial gri-
mace. The patient experiencing chronic pain may look sad
but rarely looks acutely ill. The adolescent might state, “No
one believes that I am in pain because I look normal.” A
comment to facilitate the therapeutic alliance might be, “I
am sorry you are feeling so much pain, but maybe I can
help.”

PATHOPHYSIOLOGY OF CHRONIC AND RECURRENT


ABDOMINAL PAIN

Basic and clinical research has exploded over the past 25


years, resulting in understanding of chronic and recurrent
abdominal pain. Chronic pain is often caused by a sensiti-
Figure 1. Functional gastrointestinal disorders dominate diagnoses in
Norwegian children with abdominal pain. FAP¼functional abdominal
zation of primary visceral afferent nerves to pressure and
pain. IBS¼irritable bowel syndrome. stretch (primary hyperalgesia) and amplification of pain

378 Pediatrics in Review


messages by deep brain nonspecific arousal systems (sec- changes in cortical thickness and white matter pathways
ondary hyperalgesia) (6). When the pain impulse in the associated with chronic pain. (13) CNS amplification of
arousal center is strong enough, pain messages ascend to peripheral pain and nausea signals is a feature of chronic
perception in the sensory cortex on the anterior and mid- pain physiology. Finally, the disability associated with FGIDs
cingulate gyrus (Fig 2). In individuals with healthy coping may be related to a child’s prediction error or catastroph-
skills, signals from frontal lobes in the brain may reduce ization; that is, the child believes that the symptoms are
pain transmissions from the arousal centers in the midbrain severe and hopeless. In such cases, patients may exaggerate
to the sensory cortex. In contrast, past pain experiences, symptoms and believe that they cannot cope. (14) Improving
poor coping skills, expectations for pain, anxiety, and depres- children’s self-efficacy, their belief that they can help them-
sion transmissions from the frontal lobes may increase pain selves get better, may be an important factor in symptom
transmission from the deep brain arousal centers and heighten resolution.
pain perception in the sensory cortex. Children with poor Early childhood acute pain events predispose to the later
coping skills, academic or social stress, or a coexisting mental onset of chronic and recurrent abdominal pain. An event as
health disorder may be at risk of disability from abdominal innocuous as passing a nasogastric catheter in the delivery
pain. Treatment for chronic pain may target peripheral or CNS room to aspirate gastric contents was associated with a two-
mechanisms or both. fold increase in hospitalizations for abdominal pain during a
Chronic abdominal pain includes a CNS component, child’s first decade. (15) Other early life stressors associated
characterized by altered pain pathways and structural with development of FGIDs include cow milk protein hyper-
changes in the brain. (6)(7)(8) Although evidence exists sensitivity, pyloric stenosis, urinary tract infections, (16) Henoch-
about how food, infection, inflammation, intestinal per- Schönlein purpura, and umbilical hernia repair. (17) Acute
meability, and the microbiome all contribute to trigger- bacterial gastroenteritis is followed by functional abdominal
ing symptoms, pain- or nausea-associated FGIDs often pain in approximately 30% of children. (18)
respond better to treatments targeting the CNS than Chronic pain in parents also increases the risk of func-
drugs working outside the brain. Hypnosis, (9) cognitive tional abdominal pain in their offspring. Finally, func-
behavioral therapy, (10) and citalopram (11) trials in chil- tional abdominal pain may occur together with disease.
dren were more successful than the promotility drugs For example, a teenager with Crohn disease may have
cisapride and tegaserod, the gastric acid inhibitor famotidine, coexisting IBS. Despite histologic remission of the Crohn
or the antibiotic used for bacterial overgrowth, rifaximin, disease, the patient still may suffer with abdominal pain
(12) for IBS. associated with diarrhea that is relieved by defecation.
Efferent and afferent nerves connect brain and gut, each
modulating the other, which provides a rationale for the use
FUNCTIONAL GASTROINTESTINAL DISORDERS
of psychotropic drugs for abdominal pain treatment. More-
over, recent evidence from brain imaging demonstrated Most abdominal pain is associated with one or more FGIDs.
Functional symptoms are not imaginary or feigned, but they
occur without easily discovered pathology. For example, a
runner’s cramp may cause intense pain and shivering when
cold is an uncomfortable sensation, but both of these
functional symptoms are within the expected range of
normal body functioning. In gastroenterology, functional
disorders are defined by symptom-based diagnostic criteria.
The first diagnostic criteria for pediatric FGIDs were created
in the late 1990s, when a group of international thought
leaders met in Rome to create them. Because the meeting
was in Rome, the diagnostic criteria are known as Rome
criteria. The first pediatric Rome criteria were based on the
personal experiences of the experts, but the publication of
criteria stimulated an explosion of research, such that
Figure 2. Afferent chronic pain pathways. From Hussain SZ, Hyman PE. scientific evidence is accruing about the validity, epidemi-
Psychotropic medications for pediatric functional gastrointestinal
disorders. J Pediatr Gastroenterol Nutr. 2014;59(3):280-287. Reprinted with
ology, pathophysiology, and treatment of FGIDs. Some of
permission from Wolters Kluwer Health. the original diagnostic criteria have been modified based on

Vol. 37 No. 9 SEPTEMBER 2016 379


new data. The Rome 4 criteria for pediatric abdominal pain other discomfort such as nausea may be due to peptic
disorders are listed in Table 1. (19) Rome criteria replaced disease, eosinophilic gastroenteritis, or giardiasis. Endos-
Apley’s concept of recurrent abdominal pain, (1) which had copy may be necessary to assess for mucosal disease in
been the standard for 50 years. dyspepsia. However, 85% of those undergoing endoscopy
In the past, medical schools taught that every symptom for dyspepsia have no mucosal disease and are diagnosed
had a cause, and the physician’s job was to find and fix the with functional dyspepsia. (22) Because most children with
cause for pain. Children with chronic or recurrent abdom- dyspepsia have no organic disease, an acceptable approach
inal pain were thoroughly evaluated, even overinvestigated, is to educate the family about dyspepsia and initiate a
for disease. (20) Most of these children were without ob- therapeutic drug trial with a proton pump inhibitor, cypro-
jective pathology and were labeled as having “recurrent heptadine, or tricyclic antidepressant. If symptoms resolve
abdominal pain.” Families and physicians were often frus- in the subsequent month, there is no need for endoscopy. If
trated and dissatisfied with each other when there was no symptoms persist, an endoscopy and biopsies may be
answer as to what was causing the pain other than excluding undertaken.
serious disease. Families rejected the idea that the pain was Testing should be limited and focused to rule out a
“all in her head.” An alternative to the traditional medical specific diagnosis. Testing is needed when the child pre-
model is the biopsychosocial model, which recognizes that sents with alarm features (Table 2) that suggest disease.
symptoms may be influenced by disease, psychological ill- Testing may be necessary to assuage caregiver fears. On the
ness, developmental challenges, social factors, genetics, and other hand, each negative result may reinforce caregiver
functional disorders. Instead of the dualistic approach that belief that something is being missed.
implies if there is no disease of the body, then the pain is
imaginary, the biopsychosocial model integrates mind-body
DIFFERENTIAL DIAGNOSIS
interactions into an understanding of a whole person. Symptom-
based diagnostic criteria are symptom clusters that occur School Phobia and Separation Anxiety
with regularity in a population with functional abdominal Physical symptoms may arise from emotional distress.
pain. Validation studies have proved that the symptom-based School phobia and separation anxiety are two common
criteria “breed true.”(21) entities that may present with abdominal pain. Clinicians
An important advantage of symptom-based diagnostic must obtain a history of when the pain episodes occur and
criteria is that a clinician may arrive at a diagnosis on the how long they last for both conditions. With school phobia,
first visit. For example, the clinician may say, “You have had symptoms are most prominent in the mornings before
abdominal pains for several hours almost every day for the school and improve as the day progresses. Symptoms
past 3 months that are associated with straining to defecate may be absent on weekends. The clinician should ask about
and infrequent passage of hard stools. There has been no bullying and assess the patient’s sense of academic and
blood in your stools, no fevers, and no weight loss. Therefore, social competence. “What is harder for you at school: doing
you meet criteria for a diagnosis of constipation-predominant the schoolwork or getting along with the other kids?” Once
irritable bowel syndrome. We know what it is. It is not dan- the source of stress is identified, it can be managed. Com-
gerous. It comes and goes. There are options for treatment.” munication and collaboration among school, parents, and
A second advantage of symptom-based diagnosis is a reduc- clinician are helpful in resolving school phobia.
tion in the costs for evaluating the symptoms. (20) Instead of Separation anxiety disorder is an anxiety disorder of
ruling out disease with laboratory tests and endoscopy, it is middle childhood (peak onset between 7 and 9 years) that
possible to make a positive diagnosis with symptom-based is characterized by developmentally inappropriate and exag-
criteria. The clinician can explain that the patient’s symptoms gerated fear of separation. Gastrointestinal symptoms of
are diagnostic for an FGID, but if symptoms fail to improve periumbilical pain, nausea, and vomiting intensify when
with treatment, new symptoms develop, or symptoms change separation from the caregiver is imminent. Situations
in character, the clinician provides reassurance that he/she is involving loss, such as moving to a new home or school,
available and always willing to re-evaluate. divorce, or death of a family member; stressors such as
academic challenges and difficult peer relationships; and an
overprotective parenting style increase the risk for onset or
TESTING
exacerbation of separation anxiety. Children with separation
Sometimes testing is necessary to assess for disease. For anxiety may develop symptoms and refuse to go to school or
example, dyspepsia defined by upper abdominal pain or camp or spend any time away from their major attachment

380 Pediatrics in Review


TABLE 1. Rome Criteria for the Pediatric Pain-associated Functional
Gastrointestinal Disorders (15)*
FUNCTIONAL DYSPEPSIA (FD)

Must include 1 or more of the following bothersome symptoms at least 4 times a month for at least 2 months prior to diagnosis:
1. Postprandial fullness
2. Early satiation
3. Epigastric pain or burning not associated with defecation
4. After appropriate evaluation, the symptoms cannot be fully explained by another medical condition
Within FD, the following subtypes are now adopted:
1. Postprandial distress syndrome includes bothersome postprandial fullness or early satiation which prevents finishing a regular meal.
Supportive features include upper abdominal bloating, postprandial nausea, or excessive belching.
2. Epigastric pain syndrome includes all of the following: bothersome (severe enough to interfere with normal activities) pain or burning
localized to the epigastrium. The pain is not generalized or localized to other abdominal or chest regions and is not relieved by defecation or
passage of flatus. Supportive criteria can include: a) burning quality of the pain but without a retrosternal component and b) commonly
induced or relieved by ingestion of a meal but may occur while fasting.
IRRITABLE BOWEL SYNDROME (IBS)
1. Abdominal pain at least 4 days per month over at least 2 months associated with 1 or more of the following:
a. Related to defecation
b. A change in frequency of stool
c. A change in form (appearance) of stool
2. In children with constipation, the pain does not resolve with resolution of the constipation (children in whom the pain resolves have
functional constipation, not IBS)
3. After appropriate evaluation, the symptoms cannot be fully explained by another medical condition
Criteria fulfilled for at least 2 months prior to diagnosis
ABDOMINAL MIGRAINE
Must include all of the following occurring at least twice:
1. Paroxysmal episodes of intense, acute periumbilical, midline, or diffuse abdominal pain lasting 1 hour or more (should be the most severe
and distressing symptom)
2. Episodes are separated by weeks to months
3. The pain is incapacitating and interferes with normal activities
4. Stereotypical pattern and symptoms in the individual patient
5. The pain is associated with 2 or more of the following:
a. Anorexia
b. Nausea
c. Vomiting
d. Headache
e. Photophobia
f. Pallor
6. After appropriate evaluation, the symptoms cannot be fully explained by another medical condition
Criteria fulfilled for at least 6 months prior to diagnosis
Continued

Vol. 37 No. 9 SEPTEMBER 2016 381


TABLE 1. (Continued )

FUNCTIONAL ABDOMINAL PAIN – NOT OTHERWISE SPECIFIED


Must be fulfilled at least 4 times per month and include all of the following:
1. Episodic or continuous abdominal pain that does not occur solely during physiologic events (eg, eating, menses)
2. Insufficient criteria for IBS, FD, or abdominal migraine
3. After appropriate evaluation, the abdominal pain cannot be fully explained by another medical condition
Criteria fulfilled for at least 2 months prior to diagnosis
FUNCTIONAL CONSTIPATION
Must include 2 or more of the following occurring at least once per week for a minimum of 1 month with insufficient criteria for a diagnosis of IBS:
1. Two or fewer defecations in the toilet per week in a child of a developmental age of at least 4 years
2. At least 1 episode of fecal incontinence per week
3. History of retentive posturing or excessive volitional stool retention
4. History of painful or hard bowel movements
5. Presence of a large fecal mass in the rectum
6. History of large-diameter stools that can obstruct the toilet
7. After appropriate evaluation, the symptoms cannot be fully explained by another medical condition

*More information is available at http://theromefoundation.org.

figure. Many cases resolve with simple interventions from the diagnosis. There may be a history of pain with movement
pediatrician using a rehabilitation approach targeting 3 goals: but not with eating or defecation. Stretching exercises,
1) reducing fear and avoidance of separation, 2) increasing the bending, and rotating the trunk may elicit the pain. Carnet’s
child’s participation in age-appropriate activities, and 3) re- test is said to discriminate anterior cutaneous nerve entrap-
directing attention to the child’s competent behavior (partici- ment syndrome from visceral pain. (24) After localizing an
pating in school activities, helping others). (23) area of maximal tenderness by palpation, the clinician
The goal of management for both school phobia and applies pressure to the tender spot and asks the patient
separation anxiety is to help the child learn positive methods to tense the abdominal muscles by lifting head or legs. The
of coping with and transcending the fear. Caregivers should examiner releases pressure on the abdomen and reapplies
receive clear instructions from the clinician on how to get pressure on the same spot. If the pain is from the viscera, the
the child back to school. The longer the child is out of school, reapplication of pressure fails to cause intense pain. When
the more difficult becomes the return. the pain is from cutaneous nerve entrapment syndrome,
pain upon reapplication of pressure is equivalent to that of
Functional Constipation the first palpation. Abdominal wall pain is usually a benign
Sometimes there may be overlap between constipation- condition, but if persistent, it may be treated by a pain
predominant IBS (c-IBS) and functional constipation (Table 1). specialist with injections of local anesthesia and corticoste-
Many patients have abdominal pain and constipation. If the roids. (25)
patient says that the constipation is worse than the pain, the
diagnosis is functional constipation. If pain is the most Celiac Disease and Non-celiac Gluten Sensitivity
important feature of the illness, the diagnosis is c-IBS. Celiac disease occurs in about 1 in 180 individuals, although
the ratio of symptomatic-to-asymptomatic patients is 1:7.
Abdominal Wall Pain The most common presenting complaint in childhood
Approximately 1 in 30 children presenting with bellyaches celiac disease is abdominal pain. (26) Screening for celiac
has abdominal wall pain. Chronic abdominal wall pain disease with serologic testing is reasonable in many children
is frequently confused with visceral pain, often leading with abdominal pain. Celiac disease is not more frequent in
to extensive diagnostic testing before reaching a correct patients with FGIDs than it is in the general population.

382 Pediatrics in Review


Lactase Deficiency
TABLE 2. Alarm Signs and Symptoms Carbohydrate malabsorption causes pain because the osmotic
Prompting Testing for Disease load of luminal carbohydrate draws in water from the intes-
tinal walls, thus increasing the luminal volume, stretching the
• Pain localized to the right upper or right lower quadrants
bowel wall, and activating stretch and pain receptors located
• Blood in the stools
in the bowel wall. Fermentation of unabsorbed carbohydrate
• Weight loss in the colon results in gas and another osmotic load, drawing
• Slow or delayed growth and delayed puberty more water into the lumen and stretching the colon wall,
• Odynophagia causing pain and diarrhea.
Certain ethnic groups (everyone except Northern Euro-
• Dysphagia
peans) may develop a relative lactase deficiency in the late first
• Persistent vomiting or second decade. If milk causes pain, substitutes such as soy
• Family history of inflammatory bowel disease or celiac disease milk do not cause symptoms. Lactase deficiency may be diag-
• Fevers nosed by a breath hydrogen test using lactose as a substrate.

• Arthritis
• Perianal disease: skin tags, fissures, fistulae
Sucrase Deficiency
Less common than lactase deficiency in late childhood is con-
genital sucrose deficiency, which may cause pain and other
Some patients report symptoms caused by wheat and/or
symptoms indistinguishable from IBS. (28) Sucrase deficiency
gluten ingestion, even though they do not have celiac
may be detected by a breath test using sucrose as a substrate.
disease or wheat allergy, as indicated by negative celiac
disease serologies and immunoglobulin E-mediated assays.
Most of these patients report both gastrointestinal and non- Fructose Malabsorption
gastrointestinal symptoms, and all report symptom im- Fructose malabsorption is less common than lactase defi-
provement with a gluten-free diet. This condition has ciency and is a cause of diarrhea and gas. (29) Fructose
been named non-celiac gluten sensitivity. The clinical response deficiency may be diagnosed with a breath test using fruc-
to a gluten-free diet may be caused by a variety of mechanisms, tose as the substrate.
including placebo effect, FODMAP (fermentable oligo-
di-monosaccharides and polyols) reduction, and gluten sensi- Biliary Dyskinesia
tivity. Thus, a diagnosis of non-celiac gluten sensitivity should Biliary dyskinesia is defined in most pediatric studies by 3
be approached with caution and not be based on a short-term criteria: chronic upper abdominal pain, delayed excretion
improvement in symptoms on a gluten-free diet. of radionuclide on a gallbladder emptying study, and ab-
sence of gallstones. Biliary dyskinesia is treated by chole-
Peptic Ulcer Disease and Helicobacter pylori Infection cystectomy. A growing trend in many communities is to
Peptic ulcer disease is uncommon during childhood, with refer children and adolescents to surgeons for evaluation
prevalence estimates of 1 in 2,000 children. Ulcers may present of chronic or recurrent upper abdominal pain. However,
with abdominal pain, although 50% of childhood ulcers present chronic and recurrent abdominal pain is not the same as
with a complication of bleeding, obstruction, or perforation. biliary colic, the intense episodic pain required for a biliary
Gastric H pylori infection is the most common bacterial dyskinesia diagnosis in adults. Moreover, there is no evidence
infection worldwide. It is most prevalent in low-income that hepatobiliary iminodiacetic acid (HIDA) scan results are
populations. H pylori is associated with acute and chronic valid or reliable for diagnosing biliary dyskinesia in children.
gastritis, duodenal ulcer disease, and increased risk for gas- In adults, IBS and functional dyspepsia are associated with
tric cancer. H pylori antigens in a stool sample are 95% delayed gallbladder emptying, but there are no data in chil-
accurate for diagnosis. It is prudent to eradicate the infec- dren. Pediatric studies of biliary dyskinesia use symptoms
tion with a course of acid suppression and multiple antibi- consistent with functional dyspepsia to diagnose biliary dys-
otics. However, H pylori may be an incidental finding in kinesia. Leaders in the pediatric gastroenterology commu-
patients with functional abdominal pain, and eradication of nity stress that dyspepsia responds to nonsurgical treatment.
H pylori may not result in pain reduction. There are evi- In the author’s opinion, clinicians should avoid screening
dence-based guidelines for assessing and treating H pylori in for biliary dyskinesia and instead treat chronic and recurrent
children. (27) upper abdominal pain or nausea as dyspepsia.

Vol. 37 No. 9 SEPTEMBER 2016 383


TREATMENT FOR FUNCTIONAL ABDOMINAL PAIN excellent safety profile of the drugs and the need to relieve
concerns that the pain may be caused by esophagitis,
Treatment for abdominal pain varies with the style of the
gastritis, duodenitis, or peptic ulcer disease. Narcotics that
clinician, patient/caregiver preferences, and availability of
are a mainstay of treatment for acute pain, including abdom-
the variety of modalities. The clinician should establish a
inal migraine, should not be used for chronic and recurrent
therapeutic alliance with the family before the family will
abdominal pain. Aspirin, ibuprofen, and other nonsteroidal
accept a symptom-based diagnosis and agree to suspend the
anti-inflammatory drugs should not be prescribed for ab-
search for disease. Treatment of functional abdominal pain
dominal pain because they may disrupt the gastric mucosal
disorders should always include reassurance, empathy, and
barrier and cause gastritis and ulcers. A recent systematic
education. Children and families should be assured that the
review of pediatric randomized, placebo-controlled drug
clinician believes the pain is real. The clinician explains that
trials for functional abdominal pain (33) found 8 studies, low
in children, abdominal pain without disease is more common
numbers of study participants, and many limitations and
than abdominal pain with disease. Although pharmacologic
biases. Meta-analyses suggest that tricyclic antidepressants
treatment is common, cognitive behavioral therapy (CBT) and
and serotonin reuptake inhibitors are effective for adults with
hypnotherapy have been shown to be effective for abdominal
abdominal pain. Data supporting anticholinergic, “antispas-
pain. The success of psychological interventions as well as
modic” drugs are less robust. Overall, the quality of evidence
the powerful effect of placebos demonstrates the complemen-
for pharmacologic therapy in children with chronic or recur-
tary influences between the brain and the gut and the impor-
rent abdominal pain is low to very low.
tance of the CNS in the pathophysiology of abdominal pain.
In randomized clinical trials, responses to placebos for
functional abdominal pain are usually about 40%. (34) For
Education
some clinicians who are learning what works for their patients,
Making a symptom-based diagnosis and providing a lucid
a 40% response rate may condition them to continue to pre-
explanation for symptoms is usually sufficient to reduce
scribe drugs that are essentially no better than placebos. (35)
the spoken and unspoken fears that prompted the visit.
These clinicians’ learned responses might be slow to extinguish
Patients and families want to know the answers to 4 ques-
because a 40% response rate reinforces their prescribing habits.
tions: What is wrong? Is it dangerous? Will it go away? What
can we do about it? The clinician should state the answers
to each question clearly. In gastroenterology clinic, 9% of fam-
Psychological Treatment
A recent review of nonpharmacologic treatment of func-
ilies are satisfied with a clear diagnosis and a promise of con-
tional abdominal pain concluded that hypnosis and CBT
tinuing availability, and they decline other treatment. (30)
were more effective than placebo for pediatric abdominal
Diet Changes pain. (36) However, limited access to mental health services
A minority of patients identifies specific foods as a source is common in many communities. Identifying the stressors
for pain, such as fried foods, red sauces, and milk. Patients triggering symptoms and working to eliminate such stres-
should eliminate the offending foods from their diets. There sors may be a key to successful management.
is no evidence that a lactose-, gluten-, histamine-, or carbonic One study found that CBT reduced pain better than an
acid-free diet; increased fluid intake; or prebiotics have effects identical time for education and support (attention control).
on pain-associated FGIDs. Probiotics have generally been In this trial, parents and children participated in 3 brief
ineffective for abdominal pain with 2 exceptions in adult weekly sessions that focused on relaxation training, address-
studies. Probiotics VSL#3 (31) and Lactobacillus GG (32) may ing inappropriate thoughts about symptoms, and working
be effective in some cases of abdominal pain associated with with parents to modify the family’s responses to child illness
bloating and diarrhea. Extra fiber is not helpful in most cases. behaviors (social learning). (10)(37)
Hypnotherapy is effective for abdominal pain. In a ran-
Pharmacologic Treatment domized, controlled trial of 52 children, pain intensity,
There are no drugs approved by the U.S. Food and Drug frequency, and somatization were reduced after treatment
Administration (FDA) for treatment of abdominal pain in and persisted after 5 years, indicating long-term benefit.
children, but that does not hamper prescribing. Proton pump (9)(38) Self-hypnosis through audiotapes was effective and
inhibitors and histamine-2 receptor antagonists appear to be is a readily available, cost-effective technique. (39)
the most prescribed drugs, despite symptom improvement There is weak evidence to recommend complementary
rates being similar to those for placebo. The rationale for and alternative interventions such as acupuncture or yoga
prescribing gastric acid suppression may relate to the for abdominal pain.

384 Pediatrics in Review


Abdominal Migraine antagonists and proton pump inhibitors can be offered to
Abdominal migraine is 1 of 3 paroxysmal brain/gut disor- older children and adolescents for pain-predominant
ders, along with cyclic vomiting syndrome and migraine symptoms. (42) If cure of functional dyspepsia is defined
headaches. Paroxysmal brain/gut disorders are named by as complete symptomatic relief after 4 weeks of treat-
the dominant symptom: abdominal pain, vomiting, or head- ment, omeprazole is superior to ranitidine, famotidine,
ache. Abdominal migraine accounts for about 0.5% of ab- and cimetidine. (43) Although convincing data are lacking
dominal pain patients in a pediatric gastroenterology clinic, in children, low-dose tricyclic antidepressant therapy with
making it the least common of the pain-associated FGIDs. A agents such as amitriptyline and imipramine is often
most important aspect of abdominal migraine is recogniz- considered in difficult cases.
ing the symptom-based criteria to make a proper diagnosis. A recent trial in adults showed amitriptyline to be
Often children with abdominal migraine visit emergency de- superior to placebo and citalopram in the epigastric pain
partments, are evaluated for acute abdomen, and receive no subtype but not the postprandial distress subtype. (44)
diagnosis and no relief from pain because emergency depart- Extrapolating these data to pediatrics might be risky,
ment clinicians do not want to mask symptoms. The child but one approach might be to treat epigastric pain with
suffers needlessly. In the author’s opinion, a diagnosis of acute amitriptyline and postprandial distress with another
abdominal migraine frees the clinician to titrate narcotics to drug. Two drugs that improve gastric receptive relaxation
restful sleep, with repeat doses as needed until the episode and treat nausea and early satiety are buspirone and
passes. After the episode is over, future episodes are usually mirtazapine. Buspirone, an anxiolytic, requires multiple
prevented with amitriptyline 1 mg/kg per day. dosing daily but has no black box warning about suicide.
(45) Mirtazapine, an antidepressant, requires only a
single bedtime dose and facilitates restful sleep in addi-
DRUG TREATMENT FOR FUNCTIONAL DYSPEPSIA
tion to treating postprandial distress, anxiety, panic, and
(TABLE 3)
depression. (46) For chronic daily or constant nausea,
There are no FDA-approved drugs for children with func- mirtazapine is superior to antiemetics such as prome-
tional dyspepsia. Cyproheptadine is commonly used for dys- thazine or ondansetron.
pepsia in children. (40) In another trial, cyproheptadine
reduced the episodes and severity of functional abdominal
TREATMENT FOR IRRITABLE BOWEL SYNDROME
pain. (41)
There are no prospective randomized treatment trials for The Low FODMAP Diet
pediatric functional dyspepsia. Extrapolation from adult The FODMAPs include fructose (fruits, honey, high-
data suggests that acid blockade with histamine receptor fructose corn syrup), lactose (dairy), fructans (wheat, garlic,

TABLE 3. Drug Treatment for Functional Dyspepsia


FUNCTIONAL DURATION OF
DISORDER DRUGS DOSE MEDICATION CLASS TREATMENT ADVERSE EFFECTS COMMENTS

Functional Epigastric pain: 10-50 mg qhs Tricyclic Until the pain goes Constipation, sedation; Start at 10 mg and increase
Dyspepsia Amitriptyline antidepressant away plus 6 months nightmares if patient by 10 mg/wk until pain
misses a dose resolves or 50 mg
Postprandial distress: 0.25-0.5 mg/kg per day Histamine-1 and Until the discomfort goes Fatigue, dizziness, Increased appetite in
Cyproheptadine divided bid or tid serotonin-1 away plus 6 months weight gain about 50% of children
antagonist lasts about 3 weeks

Irritable Bowel d-IBS: Amitriptyline 10-50 mg qhs Tricyclic Until the pain goes away Constipation, sedation, ECG unnecessary in
Syndrome antidepressant plus 6 months nightmares if patient individuals with
misses a dose no heart disease
Alosetron 0.5-1 mg qd or bid Serotonin-3 receptor Indefinite Constipation Very effective; expensive
antagonist
c-IBS: Imipramine 10-50 mg qhs Tricyclic Until the pain goes away Sedation Imipramine is less
antidepressant plus 6 months constipating and sedating
than amitriptyline
Lubiprostone 8-24 mg qd or bid Chloride channel Indefinite Diarrhea, nausea Expensive
activator

bid¼twice daily, c-IBS¼constipation-predominant irritable bowel syndrome, d-IBS¼diarrhea-predominant irritable bowel syndrome,
ECG¼electrocardiography, qd¼daily, qhs¼every bedtime, tid¼thrice daily

Vol. 37 No. 9 SEPTEMBER 2016 385


for treating IBS. (48) Dosing is 1 capsule for symptoms 1 to
3 times daily. Bovine serum immunoglobulin taken twice
daily was effective for diarrhea-predominant IBS (d-IBS)
in adults. (49)

Drugs (Table 3)
There are no FDA-approved drugs for treatment of chil-
dren with IBS. Randomized clinical trials are missing for
most drugs used for childhood IBS. For example, despite
their routine use by clinicians, there is no evidence that
acid suppression with histamine-2 receptor antagonists
Figure 3. Preteens and teens with functional symptoms associated with or proton pump inhibitors is helpful in IBS.
disability fall into a gap between conventional medicine and Tricyclic antidepressants are effective treatment for
conventional mental health. They require a clinician who understands
functional symptoms and a rehabilitation approach to treatment. IBS in adults, (50) but 2 randomized, controlled trials in
children were unsatisfying because placebo-treated chil-
onion, inulin), galactans (legumes such as beans, lentils, dren did as well as amitriptyline-treated children. (51)(52)
soybeans), and polyols (sweeteners containing isomalt, man- However, amitriptyline dosing in the pediatric trials may
nitol, sorbitol, xylitol, stone fruits such as avocados, apricots, have been too low to achieve a response. Amitriptyline and,
cherries, nectarines, peaches, plums). FODMAPs may not be to a lesser extent, imipramine are sedating and consti-
digested or absorbed well and could be fermented by bacteria pating. If the patient has abdominal pain, diarrhea, and
in the intestinal tract when eaten in excess. Typical IBS insomnia, a tricyclic antidepressant may be a good choice
symptoms of diarrhea, constipation, gas, and bloating may (Table 3).
improve with a low FODMAP diet. (47) A low FODMAP diet In children with d-IBS who are unable or unwilling
is best initiated in consultation with a dietitian to assure that to take amitriptyline because of concerns about cardiac
the diet meets the child’s nutritional needs. arrhythmias, seizures, or mood effects, the serotonin-3
receptor antagonist alosetron is effective. Clonidine may
Medical Foods be a good choice for d-IBS and disordered sleep.
Two medical foods are on the market to treat IBS. IBGast is There are several alternatives for children with c-IBS.
peppermint oil in an enteric-coated capsule. In one pediatric A clinician might choose to treat the constipation with
randomized, controlled trial, peppermint oil was effective polyethylene glycol titrated to soft stools together with a

Figure 4. Pathogenesis of pain-associated


disability syndrome (PADS). PADS may be
triggered by factors causing autonomic
arousal in a preteen or teen with a functional
gastrointestinal disorder. From Hyman PE,
Bursch B, Sood M, Schwankovsky L, Cocjin J,
Zeltzer LK. Visceral pain-associated disability
syndrome: a descriptive analysis. J Pediatr
Gastroenterol Nutr. 2002;35(5):663-8.
Reprinted with permission from Wolters
Kluwer Health.

386 Pediatrics in Review


tricyclic antidepressant or serotonin reuptake inhibitor for on pain, and fewer opportunities for distraction and social
pain. Lubiprostone binds to small intestinal epithelium, interaction. In addition, children who stay at home for long
activating chloride channels to facilitate secretion of chlo- periods of time may become physically deconditioned and
ride and water. Lubiprostone is approved by the FDA for c- less secure in their ability to interact with their peers.
IBS and functional constipation in women, but there are Children who stay at home sometimes adopt a sick role that
scant data in children. (53) Adherence may be better with 1 makes rehabilitation difficult.
or 2 tablets of 24-mg lubiprostone per day compared to Pain-associated disability syndrome (PADS) is defined
asking a teenager to mix and drink polyethylene glycol. as a preteen or teen with at least 2 months of abdominal
Lubiprostone should be taken with food to reduce the pain or other symptom such as nausea that is unresponsive
chances of nausea. to acute management strategies. (54) Doctors do many
tests but find no disease, yet symptoms interfere with the
patient’s routine activities, such as attending school, eat-
WHEN TO SEE A SPECIALIST
ing, and sleeping. The symptoms and disability are out of
Children should be referred to a pediatric gastroenterologist proportion to the clinicians’ findings. Physicians some-
for further testing and/or treatment in the event of: times admit that they find nothing wrong and refer
• Treatment failure families to mental health professionals. Mental health
• Prolonged school absence professionals usually find no overt psychopathology dur-
• Presence of alarm features ing the first interview because the patient converts emo-
• Abnormal laboratory test results (including abnormal tional distress into physical symptoms and denies
celiac tests) symptoms of a mood disorder. PADS patients fall into
a gap between conventional medicine and conventional
mental health (Fig 3).
DISABILITY ASSOCIATED WITH FUNCTIONAL
PADS appears to be a downward spiral of disability and
ABDOMINAL PAIN
pain that has 3 continuously interacting features: abnormal-
Children with abdominal pain may miss school. Care- ities in gastrointestinal motility, abnormalities in sensory
givers should understand that staying home from school experience such as pain and nausea, and dysregulation in
is considered a disability in the same way that staying the autonomic nervous system (Fig 4).
home from work is considered a disability for adults. The Every PADS patient meets diagnostic criteria for 1
more missed school, the more difficult it is to return. or more FGIDs that are diagnosed with symptom-based
Staying home leads to greater disability, increased focus criteria. FGIDs, which occur in approximately 25% of the

Figure 5. Multidisciplinary biopsychosocial


rehabilitation model for treating disability
associated with functional gastrointestinal
disorders. From Hyman PE, Bursch B, Sood M,
Schwankovsky L, Cocjin J, Zeltzer LK. Visceral
pain-associated disability syndrome: a
descriptive analysis. J Pediatr Gastroenterol
Nutr. 2002;35(5):663-8. Reprinted with
permission from Wolters Kluwer Health.

Vol. 37 No. 9 SEPTEMBER 2016 387


population, are associated with minor abnormalities in
gastrointestinal motility and sensation, but nearly all Summary
affected patients cope with such disorders without
• On the basis of strong research evidence, most chronic and
becoming disabled. In PADS, a developmental, family, recurrent abdominal pain in children and adolescents is
or mental health problem causes sustained autonomic functional, meaning that symptoms are not feigned, but there is
arousal. The intensity of symptoms and the degree of no easily detected disease.
disability are proportional to the severity of emotional • Symptom-based diagnostic criteria facilitate rapid diagnosis for
distress. PADS severity is inversely correlated with most children and adolescents with functional abdominal pain.
an individual’s assessment of his or her own academic For most children who meet diagnostic criteria for a functional
disorder and have no warning signs for disease (weight loss,
or social competence. PADS patients often feel helpless.
fevers, blood in stool), no testing is necessary or desirable.
Catastrophization, the belief that symptoms can only
• On the basis of strong epidemiologic evidence, irritable bowel syndrome
worsen and the patient is helpless, further activates auto- (IBS), defined by chronic or recurrent abdominal pain associated with
nomic arousal. Similarly, an external locus of control, the diarrhea or constipation or alternating diarrhea and constipation, is the
belief that the suffering can be reduced only from outside most common of the functional gastrointestinal disorders.
sources, interferes with coping and amplifies arousal. Treat- • Because of the very few prospective randomized, controlled trials
ment for teens with PADS encompasses a shift away from in children, there is no generally accepted safe and effective
the medical model, in which a doctor does something to treatment for IBS in this population. On the basis of strong
research evidence, treatments for IBS in adults include tricyclic
cure a passive patient, to a rehabilitation model, in which
antidepressants and cognitive behavioral therapy.
clinicians teach the child the necessary skills to return to a
• On the basis of strong research evidence, the placebo response
normal lifestyle.
rate in functional abdominal pain is approximately 40%; this
Treatment includes physical therapy, psychotherapy, response rate ensures that any treatment works some of the time.
and drugs to treat coexisting mental health issues and • On the basis of some research evidence and consensus, disability
sleep disruptions (Fig 5). Psychotherapy and hypnosis have from a functional disorder is proportional to comorbid psychological
the advantage of effects that last long after the treatment distress from a mental health disorder or learning disability.
period compared to drug effects that last only as long as • On the basis of primarily consensus, due to lack of prospective
treatment continues. Drugs have the advantage of not studies, treatment for those disabled by functional abdominal
requiring a psychotherapist. However, strong evidence pain requires shifting from an acute medical to a rehabilitation
model and involves a team approach that includes cognitive
for drug efficacy in children with pain related to FGIDs
behavioral therapy, medication to regulate sleep and reduce
is missing. Treatment requires a multidisciplinary biopsy- autonomic arousal, and physical therapy. Refusal to engage with
chosocial team approach that must include the family. mental health treatment is associated with treatment failure.
When the child or family does not accept the diagnosis
or challenges the treatment, management fails. When the
child, family, and team of professionals are in step, treat- References for this article are at http://pedsinreview.aappublications.
ment succeeds. org/content/37/9/377.

Parent Resources from the AAP at HealthyChildren.org


Chronic and Recurrent Abdominal Pain
• https://www.healthychildren.org/English/health-issues/conditions/abdominal/Pages/Abdominal-Pain-in-Children.aspx
• Spanish: https://www.healthychildren.org/spanish/health-issues/conditions/abdominal/paginas/abdominal-pain-in-children.aspx
• https://www.healthychildren.org/English/health-issues/conditions/abdominal/Pages/Abdominal-Pains-in-Infants.aspx (English only)

388 Pediatrics in Review


PIR Quiz
There are two ways to access the journal CME quizzes:
1. Individual CME quizzes are available via a handy blue CME link under the article title in the Table of Contents of any issue.
2. To access all CME articles, click “Journal CME” from Gateway’s orange main menu or go directly to: http://www.aappublications.
org/content/journal-cme.

1. A 10-year-old girl is brought by her parents to see you for the first time for evaluation of a 1- REQUIREMENTS: Learners
year history of abdominal pain that she has been experiencing several times a week. You can take Pediatrics in
know that chronic recurrent abdominal pain is common among children across the world. Review quizzes and claim
You also know that most such pain is not caused by isolated organic disease, but rather credit online only at:
reflects a symptom sparked by a combination of overlapping physical, physiological, http://pedsinreview.org.
mental, and social factors. Which of the following characteristics of the pain will prompt
you to initiate an immediate search for an organic explanation?
To successfully complete
A. Has a consistent periumbilical distribution. 2016 Pediatrics in Review
B. Is associated with a 10-lb weight loss. articles for AMA PRA
C. Occurs primarily upon awakening. Category 1 CreditTM,
D. Occurs primarily upon going to sleep. learners must
E. Occurs several times a day, lasting less than 5 minutes for each episode. demonstrate a minimum
2. Over the past 25 years, the uncovering of a better understanding of the pathophysiology of performance level of 60%
chronic and recurrent abdominal pain has: or higher on this
A. Decreased the need for mental health counseling for the complaint. assessment, which
B. Increased the clinician’s ability to use history and physical examination to reliably measures achievement of
differentiate one functional disorder from another. the educational purpose
C. Increased the need for primary care physicians to refer patients with such pain to and/or objectives of this
gastroenterologists. activity. If you score less
D. Increased the requirement for sophisticated diagnostic laboratory testing. than 60% on the
E. Raised doubt about the role of social stress as a contributing cause. assessment, you will be
given additional
3. Since starting kindergarten, an 8-year-old boy has had frequent periumbilical pain and
opportunities to answer
nausea in the morning. He occasionally vomits. Which of the following statements would
questions until an overall
strongly suggest that he suffers from separation anxiety?
60% or greater score is
A. He has postbreakfast fullness, bloating, and belching. achieved.
B. He feels fine soon after being permitted to stay home from school.
C. His symptoms are present both on weekdays and weekends.
D. His symptoms are relieved by defecation. This journal-based CME
E. His symptoms improve at school as the day goes on. activity is available
through Dec. 31, 2018,
4. A 12-year-old girl has experienced daily periumbilical abdominal pain lasting several hours for
however, credit will be
the past 2 years. She rarely misses school. She has gained weight normally and experienced
recorded in the year in
normal menarche. She strains to defecate and passes small hard stools every 2 days, after which
which the learner
she experiences some relief from pain but no complete resolution. Treatment with polyethylene
completes the quiz.
glycol has failed to resolve the problem. Her physical examination yields normal results. Based
on her history and examination, which of the following is the most likely diagnosis?
A. Abdominal migraine.
B. Functional constipation.
C. Functional dyspepsia.
D. Irritable bowel syndrome.
E. Postprandial distress syndrome.

Vol. 37 No. 9 SEPTEMBER 2016 389


5. The caregivers of children and adolescents diagnosed with functional abdominal pain in
your practice ask you about options for treatment and their effectiveness. Assuming that you
already have established a therapeutic alliance with patients and families, provided them
with assurances that you believe that the pain is real, and offered empathy, education, and
reassurance, which of the following proposed treatment options currently promises the best
treatment outcome for children and adolescents with functional abdominal pain?
A. Acupuncture.
B. Changes in diet.
C. Cognitive behavioral therapy.
D. Intensive personal psychotherapy.
E. Prescription of U.S. Food and Drug Administration-approved medications.

390 Pediatrics in Review


Constipation, Irritability, and Poor Feeding in
1 2-month-old Boy

Nidhi A. Shah, MD,* Henry C. Lee, MD*


*Department of Pediatrics, Division of Neonatal and Developmental Medicine,
Stanford University, Stanford, CA.

PRESENTATION

A 2-month-old boy, who had been born at term, presents to the emergency
EDITOR’S NOTE
We invite readers to contribute Index of department with irritability and poor feeding. The infant had been healthy with no
Suspicion cases at: Submit and Track My temperature instability until approximately 2 weeks ago, when he became fussier
Manuscript. than usual. Yesterday he refused to breastfeed and had a decreased number of wet
diapers. His last stool was 5 days ago. His parents report that the infant is drooling
AUTHOR DISCLOSURE Drs Shah and Lee
have disclosed no financial relationships
more and appeared to choke on his secretions today.
relevant to this article. This commentary does The pregnancy was unremarkable and was completed with a normal vaginal
not contain a discussion of an unapproved/ delivery. The infant is exclusively breastfed, with good weight gain and normal
investigative use of a commercial product/
development. Family members were ill with a stomach virus 10 days ago, but the
device.
infant appeared unaffected. His only medications are probiotics, chamomile, and
an herbal supplement. Recent travel includes camping along the coastline. The
infant has not received any vaccines.
Vital signs are as follows: temperature of 36.7°C (98.1°F), heart rate of 166
beats per minute, respiratory rate of 75 breaths per minute, blood pressure of
85/43 mm Hg, and oxygen saturation of 95%. On physical examination, the boy is
grunting and has increased work of breathing, with subcostal retractions and
nasal flaring. Pupils are equal, round, and reactive to light; extraocular movements
are intact; and facial muscles appear symmetric. He has an absent gag reflex and
absent bowel sounds. Passive tone appears to be normal, and deep tendon reflexes
cannot be elicited, which may be due to agitation and difficulty of examination.
Laboratory evaluation reveals the following:
• White blood cell count 9,910/mL (9.91  109/L)
• Hemoglobin 12 g/dL (120 g/L)
• Platelet count 454  103/mL (454  109/L)
• Sodium 142 mEq/L (142 mmol/L)
• Potassium 5.4 mEq/L (5.4 mmol/L)
• Chloride 110 mEq/L (110 mmol/L)
• Serum carbon dioxide 11 mEq/L (11 mmol/L)
• Blood urea nitrogen 20 mg/dL (7.1 mmol/L)
• Creatinine 0.5 mg/dL (44.2 mmol/L)
• Glucose 55 mg/dL (3 mmol/L)
A chest radiograph yields unremarkable results, and a plain abdominal
radiograph (kidneys, ureter, bladder) shows substantial air in the bowel (Figure).

Vol. 37 No. 9 SEPTEMBER 2016 391


the hospital 5 days after treatment with completely normal
neurologic function.

The Condition
In the United States, an average 145 cases of botulism are
reported annually, of which 65% are cases of infantile
botulism. Botulism is caused by Clostridium botulinum, a
gram-positive, spore-forming bacterium that produces a
toxin which blocks cholinergic transmission on the presyn-
aptic side of ganglia and neuromuscular junctions. Although
infantile botulism is most commonly associated with inges-
tion of raw honey, most cases actually occur from the inges-
tion of spores found in dust and dirt. Of the 4 subtypes of C
botulinum, types A and B are responsible for most cases.
Type A botulism is believed to be more severe than type B.
Infantile botulism affects infants up to 12 months of age,
although the median age of onset is 3 to 4 months. Older
children are believed to be less susceptible to botulism due
to their mature gastrointestinal tracts, which can excrete the
spores before they germinate. The condition most com-
monly presents with constipation and poor feeding, fol-
lowed by progressive descending weakness and hypotonia.
Infants can rapidly decompensate because cranial nerve
Figure. Abdominal radiograph (kidneys, ureter, bladder) showing diffuse function is compromised, leading to a loss of suck and
intestinal dilation. gag reflexes and eventually respiratory failure.

The infant is intubated for airway protection. Blood and Diagnosis


urine are sent for culturing, and empiric antibiotics are Diagnosing infantile botulism can be difficult. The most
initiated. Stool tests ultimately lead to a diagnosis. sensitive test for diagnosis is identification of toxin in the
stool. Because constipation is present in almost all affected
infants, however, stool can be difficult to obtain. Serum mar-
DISCUSSION
kers are often negative.
Based on the infant’s presentation of respiratory distress
and abnormal neurologic findings, the differential diagno- Management
sis included sepsis, meningoencephalitis, inborn error of Supportive treatment is the mainstay of the therapy. Infants
metabolism, exposure to or ingestion of toxin, and congen- with respiratory failure must be intubated to protect the
ital neuromuscular disease. The findings of constipation, airway and placed on mechanical ventilation. Paralysis may
difficulty handling secretions, and absent gag reflex also progress rapidly, and heightened vigilance is required to
raised the possibility of infantile botulism. prevent sudden deterioration. During the period of ileus,
Stool tests were ordered for botulinum toxin, but samples nasogastric decompression along with parenteral nutrition
could not be obtained efficiently due to ileus. A saline ene- may be required.
ma was performed, and stool was obtained on the second at- BabyBIG is a safe and effective therapy to treat both types
tempt. The stool was sent to the state laboratory for botulinum A and B infantile botulism. It contains primarily immuno-
testing. Because of the delay in sending the stool and the globulin G that is pooled from donors. Because of the lack
high suspicion for infantile botulism, botulism immuno- of adverse effects associated with the treatment, infants
globulin (BabyBIG) was administered empirically. The infant with clinical presentations consistent with infantile botulism
showed improved spontaneous movement within hours of should be empirically treated with BabyBIG even before
treatment and was able to be extubated 2 days later. The state confirmation of the diagnosis. Systemic antibiotics should
laboratory confirmed the diagnosis of infantile botulism 1 day be discontinued because they might lead to lysis of C botu-
after treatment was initiated. The infant was discharged from linum in the intestines, resulting in further release of toxin.

392 Pediatrics in Review


In a randomized, controlled study, administration of • Infantile botulism almost always presents with feeding
BabyBIG reduced length of hospital admission, duration of difficulty and constipation, followed by a descending
intensive care, and duration of mechanical ventilation for paralysis.
treated infants compared to those who were not treated with • Botulism immunoglobulin should be administered early,
BabyBIG. even without the results of confirmatory tests.

Lessons for the Clinician


• Infantile botulism is most commonly caused by ingestion Suggested Readings for this article are at http://pedsinreview.
of spores found in dirt and dust. aappublications.org/content/37/9/391.

Parent Resources from the AAP at HealthyChildren.org


Case 1: Constipation, Irritability and Poor Feeding in a 2-month-old Boy
• https://www.healthychildren.org/English/health-issues/conditions/infections/Pages/Botulism.aspx
• Spanish: https://www.healthychildren.org/spanish/health-issues/conditions/infections/paginas/botulism.aspx

Vol. 37 No. 9 SEPTEMBER 2016 393


Acute Respiratory Distress and Facial
2 Petechiae in a 4-month-old Boy

Alpa Patel Shah, DO,* Mary Lu Angelilli, MD*


*Children’s Hospital of Michigan, Detroit, MI.

PRESENTATION

A 4-month-old twin infant boy presents to the emergency department (ED) with a
AUTHOR DISCLOSURE Drs Shah and Angelilli
have disclosed no financial relationships 10-hour history of increased work of breathing, poor feeding, and decreased
relevant to this article. This commentary does activity. Prior to arrival, the patient’s mother administered 2 albuterol treatments
not contain a discussion of an unapproved/
via nebulizer, which helped transiently. Then she took the child to his pediatri-
investigative use of a commercial product/
device. cian’s office, where he was given an additional albuterol treatment and sub-
sequently referred to the ED for persistent respiratory distress. The infant has had
2 hospital admissions for bronchiolitis. His parents deny any history of fever,
cough, wheezing, runny nose, or vomiting. His twin brother remains asymptomatic.
In the ED, the child is alert but tachypneic, with intercostal and subcostal
retractions and nasal flaring. His initial vital signs include a temperature of 37.8°C
(100°F), heart rate of 174 beats per minute, respiratory rate of 34 breaths per
minute, blood pressure of 111/53 mm Hg, and pulse oximetry of 80% in room air.
He weighs 4.8 kg (less than the third percentile). Findings on physical exami-
nation include a reddish discoloration on both sides of the neck and petechiae
on the right cheek. A subconjunctival hemorrhage is also noted in the right eye.
His lungs are clear to auscultation bilaterally, without wheezing, stridor, rales,
or crackles, despite persistent and severe retractions. Auscultation of the heart
reveals a regular rate and rhythm without a murmur. Capillary refill is less than 2
seconds. The remainder of the physical examination yields unremarkable results.

DISCUSSION

The initial evaluation for respiratory distress included chest radiography, which
was negative for any cardiopulmonary process; a capillary blood gas analysis that
revealed a pH of 7.32, PCO2 of 41 mm Hg, PO2 of 57 mm Hg, and bicarbonate of
19 mEq/L (19 mmol/L); a complete blood cell count and electrolyte panel, which
documented no findings of note; coagulation studies, which were within normal
limits; computed tomography (CT) scan of the head, which was negative for
hemorrhage or any intracranial pathology; and electrocardiography and echocar-
diography, the results of which were within normal limits. The patient progressed
to respiratory failure and ultimately required intubation and transfer to the
pediatric intensive care unit.
Due to the petechiae and discoloration around the child’s neck as well as the
right subconjunctival hemorrhage, which raised suspicion for trauma, the child

394 Pediatrics in Review


protection team (CPT) was consulted and a bone survey, and involves a powerful combination of hyperextension,
ophthalmology examination, and magnetic resonance im- compression, and/or distraction forces. Although rare,
aging (MRI) of the brain and cervical spine were re- hangman’s fractures have been reported in the setting of
commended. MRI of the brain documented soft-tissue child physical abuse. However, the mechanism of injury in
thickening at the C2/C3 level. MRI of the cervical spine abuse is unknown because the perpetrator does not always
demonstrated subluxation of C2 over C3, with an epidural disclose information regarding the abuse.
hematoma causing compression of the spinal cord and Even in the case of known child physical abuse, clinicians
edema in the lateral masses of C2 and C3. A CT scan of must distinguish a traumatic hangman’s fracture from a
the cervical spine showed bilateral pars fractures of C2 with congenital C2 spondylolysis. A few factors that point to
widening of the C2-C3 intervertebral disc spaces consistent abusive trauma are: disruption of the intervertebral discs,
with a hangman’s fracture (Figure). Ophthalmologic exam- ligamentous injury or hematoma on MRI, and the presence
ination yielded negative results, and the bone survey did not of callous formation or other evidence of healing on serial
document any additional fractures. Child physical abuse imaging. In this case, there was evidence of ligamentous
was diagnosed. injury, an epidural hematoma, widening of the C2/C3
disc space, and edema of the lateral masses. This com-
Differential Diagnosis bination, along with the concerning area of ecchymosis
The differential diagnosis in a previously healthy infant around the patient’s neck, petechiae on the cheek, and
presenting with acute respiratory failure in the presence subconjunctival hemorrhage, is worrisome and should
of bruising of the face and/or neck includes both non- immediately alert a clinician to the possibility of child
accidental and accidental trauma to the spine and/or spinal physical abuse.
cord. Nonaccidental trauma may specifically result from
strangulation injury or intentional smothering leading to Management
asphyxia. The management of a patient with a suspected hangman’s
fracture requires close collaboration with the neurosurgical
The Condition team. Cervical spine immobilization and observation are
A hangman’s fracture is a traumatic spondylolysis of C2. most important, although neurosurgical repair may be
The term originates from the fact that this injury was indicated in select cases. Physical therapy is a crucial com-
commonly seen as a consequence of judicial hangings. ponent of treatment to ensure appropriate rehabilitation,
Today, hangman’s fractures refer to bilateral C2 pars inter- and serial cervical spine images are recommended. The
articularis fractures and are usually the result of a sudden remainder of the management is case-dependent and focused
deceleration injury. This is seen in motor vehicle crashes on supportive care.

Figure. Computed tomography scan of the cervical spine. A. Anteroposterior image showing bilateral pars interarticularis fractures of C2. B. Lateral
image showing C2 pars interarticularis fracture.

Vol. 37 No. 9 SEPTEMBER 2016 395


Patient Course and Outcome off of mechanical ventilation. He was discharged to his
The CPT made a referral to child protective services and the maternal grandmother in stable condition with outpatient
patient was evaluated by a neurosurgeon. The cervical spine physical and occupational therapy services. The patient was
was stabilized using a cervical collar followed by an infant followed with serial cervical spine images, which showed
restraining device. The neurosurgery team noted weakness evidence of healing.
in the left upper extremity on their examination, for which
he was evaluated by a neurologist and treated by both Lessons for the Clinician
physical and occupational therapy services. Nasogastric tube • Child physical abuse must be considered in any infant
feedings were started to maintain adequate nutrition. who presents with unexplained bruises.
On the sixth day of admission, the patient’s father re- • Unexplained bruises on the neck should alert the clinician
quested a meeting with the CPT physician. He disclosed to the possibility of a cervical spine injury.
that he placed his thumbs around the front of the patient’s • Although uncommon, bilateral C2 pars fractures have
neck and dug his fingers into the back of the neck as he threw been reported specifically in cases of abusive head and
him onto the bed. The father stated that he became frustrated neck trauma.
because the baby was crying despite his efforts to feed and • It is important to distinguish a primary spondylolysis or
change him. The incident was promptly referred to local officials. persistent synchondrosis from a traumatic spondylolysis.
The patient continued to improve with cervical spine • Maintaining a high index of suspicion for neck injury with
immobilization, physical therapy, and occupational therapy. early cervical spine immobilization is crucial.
Within several days of placing him in a cervical collar, his
oromotor skills improved and nasogastric tube feedings Suggested Readings for this article are at http://pedsinreview.
were discontinued. He was also successfully weaned aappublications.org/content/37/9/394.

396 Pediatrics in Review


3 Sore Throat and Fever in a 4-year-old Boy

Katsuaki Kojima, MD,* Melissa Rosenberg, MD,† B. Keith English, MD*


*Departments of Pediatrics and Human Development, College of Human Medicine;

Department of Pediatrics, College of Osteopathic Medicine, Michigan State University,
East Lansing, MI.

PRESENTATION

A 4-year-old previously healthy and fully immunized boy presents with a 2-day
AUTHOR DISCLOSURE Drs Kojima,
Rosenberg, and English have disclosed no history of sore throat and a 1-day history of fever and drooling. He awakened today
financial relationships relevant to this article. with worsening sore throat, drooling, and subjective fever. He complains of pain
This commentary does not contain a
during swallowing and refuses oral intake. He has no cough, congestion, sick
discussion of an unapproved/investigative
use of a commercial product/device. contacts, or recent travel. His prenatal, natal, and postnatal histories contain no
significant information. His immunizations are up to date.
At initial presentation, his temperature is 39.6°C (103.4°F), heart rate is 160
beats per minute, respiratory rate is 22 breaths per minute, and oxygen saturation
is 95% in room air. Physical examination reveals an alert and interactive boy who
is drooling and appears uncomfortable. He is in the tripod position and has a
muffled voice but no stridor, nasal flaring, or labored breathing. His chest is clear
to auscultation bilaterally. His neck is supple and has no swelling, erythema, or
tenderness. His throat is not examined. Findings on the remainder of the physical
examination are unremarkable.
Results of laboratory evaluation include:
• White blood cell count 30,900/mL (30.9  109/mL), with 66% segmented
neutrophils, 22% banded neutrophils, 5% lymphocytes, and 7% monocytes
• Hemoglobin 13.9 g/dL (139 g/L)
• Hematocrit 39.1% (0.39)
• Platelet count 473  103/mL (473  109/L)
• Erythrocyte sedimentation rate, 8 mm/h
• C-reactive protein, 3.2 mg/L (30.5 nmol/L)
Blood is drawn for culture. Neck soft-tissue radiography leads to the diagnosis.

DISCUSSION

The soft-tissue neck radiograph reveals a “thumb sign.” After clinicians


administer ceftriaxone and dexamethasone, the patient is flown to a university
hospital. He is transported to the operating room for endotracheal intubation
under general anesthesia. His epiglottis is visualized and is erythematous
and swollen. Blood culture becomes positive for Haemophilus influenzae
type f after 40 hours. Repeat blood culture is negative. He receives 14 days of
antibiotic treatment with ceftriaxone. Immunodeficiency evaluation is not
performed.

Vol. 37 No. 9 SEPTEMBER 2016 397


Condition and worsen respiratory distress. An experienced physician
H influenzae type b (Hib) was historically the most common with skills in airway management and intubation should be
cause of acute epiglottitis. Immunization against Hib was present at all times.
introduced in 1985, although initially it was not a conjugate Occasionally, other infections related to Hib, including
vaccine and was not recommended for infants. The first Hib pneumonia, cervical adenopathy, or otitis media, may occur
conjugate vaccine was licensed in December 1987 and the concurrently with epiglottitis, but this is a rare finding.
incidence of Hib epiglottitis declined by 80% to 90% af-
ter the introduction. The incidence of immunoglobulin A Treatment
deficiency and immunoglobulin G subclass deficiency are Acute epiglottitis is a medical emergency that warrants
reported to be higher in those who had Hib vaccine failure. immediate intervention to secure the airway regardless of
Epiglottitis caused by other H influenzae types (a, f, and the degree of apparent respiratory distress. Without an
nontypeable), Streptococcus pyogenes, S pneumoniae, and Staphy- artificial airway, 6% of children die compared to less than
lococcus aureus (including methicillin-resistant strains [MRSA]) 1% of those with an artificial airway. The airway should be
now comprise a larger proportion of cases compared to the secured in a controlled setting, either an operating room or
prevaccine era. The median peak age of children with epiglottitis intensive care unit, via orotracheal intubation, nasotracheal
has increased from age 3 years to ages 6 to 12 years due to the intubation, or less frequently, tracheostomy.
change in etiology of the disease. Among the pediatric The choice of antibiotics depends on local resistance
population, the incidence of acute epiglottitis was 0.5 patterns. In general, a combination of a third-generation
cases per 100,000 children in the United States in 2006. cephalosporin and an antistaphylococcal agent against
Acute epiglottitis is defined as an acute inflammation of MRSA is recommended. Once an organism is identified by
the epiglottis and adjacent structures. The clinical presen- blood culture or culture of the epiglottis, antibiotics specif-
tation is characterized by rapidly progressing dysphagia, ically directed to the organism’s susceptibility should be
drooling, dysphonia, and distress (“the four Ds”). Other prescribed. The optimal duration of therapy is unknown,
characteristic findings include high temperature, toxic appear- although most physicians treat for 7 to 10 days. A longer
ance, and muffled speech, which is sometimes described as a course may be required in cases of bacteremia.
“hot potato” voice. The typical patient presents in the tripod Evidence to support routine use of glucocorticoids in pa-
position of sitting upright and leaning forward with a hyper- tients with epiglottitis is insufficient. Retrospective studies do
extended neck and chin thrust, which maximizes airway not show an association between glucocorticoid use and re-
patency. Of note, children with epiglottitis lack a cough or duced length of intensive care unit or hospital stay or duration
hoarse voice, which are characteristic of croup. of intubation. Such use may be beneficial in select patients
with severe disease or in patients who are difficult to extubate.
Diagnosis
The diagnosis is based upon visualization of a large, cherry Prognosis
red, swollen epiglottis or demonstration of epiglottal swell- Epiglottitis generally resolves after a few days of antibiotics,
ing (“thumb sign”) by lateral neck radiography. Blood cul- and the patient may be extubated once the inflammation is
tures and cultures obtained from the surface of the epiglottis under control. Most patients recover without residual airway
help identify the causative organism. Epiglottal cultures are problems if the airway is secured promptly and they are
positive in 30% to 70% of patients. treated with appropriate antibiotics.
When a clinician has a high index of suspicion for
epiglottitis based on clinical presentation, direct visualiza- Lessons for the Clinician
tion of the epiglottis with laryngoscopy should be performed • Acute epiglottitis is a medical emergency that warrants
by a physician skilled in intubation in a controlled setting, immediate intervention to secure the airway.
such as an operating room or intensive care unit, with • A combination of a third-generation cephalosporin and
preparation for subsequent artificial airway placement. an antistaphylococcal agent against methicillin-resistant
Laboratory testing and radiography should not be per- Staphylococcus aureus is recommended.
formed on patients in whom epiglottitis is strongly sus- • The incidence of Haemophilus influenzae type b epiglottitis
pected until the airway is secured. In this case, soft-tissue declined significantly after the introduction of vaccination.
neck radiography was performed due to the unclear cause of
his respiratory distress because he had been fully immu- Suggested Readings for this article are at http://pedsinreview.
nized. Agitation and anxiety may increase respiratory effort aappublications.org/content/37/9/397.

398 Pediatrics in Review


4 Two Black Eschars in a 6-year-old Girl

Zachary Graff, MD,* Geronimo Mendoza, MD,* Apurv Barche, MD,*


Curtis Turner, MD*
*Texas Tech University Health Sciences Center School of Medicine, Amarillo, TX.

PRESENTATION

A previously healthy 6-year-old Hispanic girl presents with a 4-week history of a


EDITOR’S NOTE
This case was selected for publication from left lower back skin lesion and a 1-week history of a left forearm lesion. Both
the 10 finalists in the 2015 Clinical Case lesions progressed from an initial red bump to ulceration with overlying black
Presentation program for Section on crust. The lesions are mildly itchy and not painful. She denies fever, weight loss,
Pediatric Trainees of the American night sweats, lymphadenopathy, and cough. She has no history of trauma,
Academy of Pediatrics (AAP). Zachary Graff,
international travel, tuberculosis exposure, pet bites or scratches, or exposure
MD, resident from Texas Tech University
Health Sciences Center School of Medicine,
to animal hides. There is no family history of cancer. Social history is notable for
Amarillo, TX, wrote this case report. her father working at a dairy farm.
Choosing which case to publish involved Physical examination reveals a temperature of 36.2°C (97.2°F), with other vital
considerations of the teaching value and signs within normal limits. The girl is smiling and cooperative. Examination of
excellence of writing but also the content the skin reveals a nontender 2.5  2-cm sharply defined ulcer with overlying black
needs of the journal. Another case will be
eschar on the left lower back with edematous and erythematous borders (Fig 1)
chosen from the finalists presented at this
year’s AAP National Conference and and a similar 1-cm lesion on the left forearm (Fig 2). There is no clinically
Exhibition and published in the September significant lymphadenopathy or hepatosplenomegaly. The remainder of the
2017 issue of Pediatrics in Review. examination yields normal results.
She is started on doxycycline for suspected cutaneous anthrax. However,
AUTHOR DISCLOSURE Drs Graff, Mendoza,
Barche, and Turner have disclosed no financial
polymerase chain reaction (PCR) for Bacillus anthracis returns negative and
relationships relevant to this article. This the doxycycline is discontinued.
commentary does not contain a discussion of A complete blood cell count shows a white blood cell count of 6,800/mL (6.8 
an unapproved/investigative use of a
109/L) with 48% neutrophils, 39% lymphocytes, 11% monocytes, and 2% eosin-
commercial product/device.
ophils. Results of other laboratory studies, including a complete metabolic panel,
erythrocyte sedimentation rate, C-reactive protein, serum uric acid, and lactate
dehydrogenase, are within reference ranges. Human immunodeficiency virus
antibodies are negative and a chest radiograph is normal. A pediatric oncologist is
consulted and recommends skin biopsies.

DISCUSSION

The differential diagnosis for this girl certainly included cutaneous anthrax due to
the characteristic progression of the lesions from erythematous painless papule to
ulceration and eventually to black eschar. However, the incidence of cutaneous
anthrax in the United States is extremely rare (1-2 cases per year reported), and due
to her lack of exposure to risk factors such as animals and animal products
and negative PCR test result for Bacillus anthracis, an alternative diagnosis was
pursued.

Vol. 37 No. 9 SEPTEMBER 2016 399


CD4. The CD4:CD8 ratio was 6:1. Forty percent of the
lymphoid cells expressed CD30. The cells also expressed
anaplastic lymphoma kinase (ALK). A high proliferation
rate was noted, with 70% of cells expressing Ki67. Mono-
clonal rearrangement for T-cell receptor and gamma re-
ceptors was demonstrated by PCR. A final diagnosis of
ALKþ anaplastic large cell lymphoma (ALCL) was made.
Once the pathology report was available, imaging
(computed tomography scan of chest/abdomen/pelvis
and bone scan) and bone marrow biopsy and aspira-
tion with flow cytometry were performed and ruled out
systemic involvement. She was started on oral metho-
trexate (20 mg/m2) weekly and topical bexarotene daily
Figure 1. A 2.5  2.0-cm sharply defined ulcer with overlying black for 4 weeks. Bexarotene is a retinoid that achieves its
eschar on the left lower back with edematous and erythematous borders. antineoplastic properties by binding retinoid X receptors.
Once bound, these receptors function as transcription
factors that regulate genes, which control cellular differ-
Also in the differential diagnosis was ecthyma. This
entiation and proliferation. A plastic surgeon performed
condition is characterized initially by grey-yellow crusted
wide local excisions of both lesions, which were reviewed
sores under which ulcers later form, a deep form of impe-
by pathology and demonstrated no residual disease. The
tigo involving the dermis. There is a predilection for the
girl remains in complete remission 2 years following
lower extremities. This is caused most commonly by the
treatment.
same bacterium causing impetigo, Streptococcus pyogenes.
Due to the absence of initial grey-yellow crust, location of
The Condition
lesions on the lower back and forearm, and more subacute
Primary cutaneous lymphomas (PCLs) are defined as non-
presentation of lesions over 4 weeks, ecthyma was a less likely
Hodgkin’s lymphomas limited to the skin. They often
diagnosis.
mimic infectious processes such as cutaneous anthrax
She underwent full-thickness incisional resection of the
and ecthyma and can be difficult to distinguish clinically.
back lesion. The evaluation was negative for organisms
PCLs present as solitary or multifocal red skin lesions that
on Gram stain, anthrax PCR, acid-fast bacilli, aerobic and
do not regress and may progress to ulceration and black
anaerobic cultures, and fungal cultures.
eschar. They may appear on any part of the body and often
Histopathology showed an atypical T-cell lymphoid
grow slowly, with a more subacute course than cutaneous
infiltrate rimming adipose tissue, with 70% of abnor-
anthrax.
mal lymphocytes expressing CD3, CD5, CD2, BCL2, and
Within this heterogeneous group of disorders is ALCL,
which is typically defined by the immune-histochemical
expression of CD30 and lack of ALK. In contrast to systemic
ALCL, which comprises 15% of pediatric non-Hodgkin’s
lymphomas, cutaneous ALCL is rarely found in children.
In addition, the presence of ALK expression is exceedingly
rare, and some have suggested that its expression should
be considered highly suspicious for underlying systemic
involvement.
Although systemic disease with cutaneous manifesta-
tions is treated aggressively with polychemotherapy, local-
ized cutaneous disease often spontaneously regresses or is
treated locally and cured with radiation or surgery alone.
This patient received a short course of systemic chemother-
apy, in part due to the relative lack of therapeutic trials
guiding treatment decisions for this disease. In addition,
Figure 2. A sharply defined 1-cm ulcer with overlying black eschar on
left forearm with edematous and erythematous borders. the presence of ALK expression in adults is suggestive of

400 Pediatrics in Review


systemic disease. This patient was found to have primary with very few reports of anaplastic lymphoma kinase (ALK)
cutaneous ALKþ ALCL, with no evidence of extracutaneous expression.
involvement, suggesting that ALK expression may not • Although the prognosis of this disease is favorable, early
be a reliable marker of systemic disease in pediatric detection is important for appropriate treatment to avoid
patients. potential systemic involvement.
• This case reinforces that ALK expression cannot be used
Lessons for the Clinician to distinguish systemic from localized cutaneous disease
• Physicians must maintain a broad differential diagnosis in children, as previously thought.
when confronted with unusual skin findings.
• Primary cutaneous anaplastic large cell lymphoma may be Suggested Readings for this article are at http://pedsinreview.
confused with infection and is a rare diagnosis in children, aappublications.org/content/37/9/399.

Vol. 37 No. 9 SEPTEMBER 2016 401


in
Brief
Pulse Oximetry and the Neonate
Philip Roth, MD, PhD*
*Staten Island University Hospital, Hofstra-North Shore/LIJ School of Medicine, Staten Island, NY.

AUTHOR DISCLOSURE Dr Roth has disclosed Evaluation of the newborn who has cyanosis requires prompt and logical as-
no financial relationships relevant to this
sessment. Although entities such as polycythemia and methemoglobinemia
article. This commentary does not contain a
discussion of an unapproved/investigative must be considered, the diagnoses that must be addressed most urgently are
use of a commercial product/device. primary respiratory disease, congenital heart disease (CHD), and persistent
pulmonary hypertension of the newborn (PPHN). These categories of disease
are not mutually exclusive, as in the case of meconium aspiration with PPHN,
but their distinct pathophysiologic mechanisms account for different results
in diagnostic testing.
The essence of PPHN lies in the pathologic failure of pulmonary artery
pressure to decline postnatally. Although pulmonary artery pressure varies
directly with pulmonary vascular resistance (PVR), pulmonary blood flow, and
pulmonary capillary wedge pressure, the first of these plays the most prominent
role in PPHN. Persistently elevated PVR may result from the perinatal failure to
elaborate nitric oxide and/or prostaglandin I2 in addition to increased production
of phosphodiesterase-3 or -5, endothelin-1, reactive oxygen species, and rho-
kinase. The net effect is right ventricular dysfunction as well as right-to-left
shunting across fetal channels (ie, the patent foramen ovale and ductus arterio-
sus), which results in decreased pulmonary blood flow. The affected infant
develops ventilation/perfusion mismatch, hypoxemia, and acidosis that can lead
to left ventricular dysfunction, decreased output, and shock. When differentiating
Critical Congenital Heart Disease Screening
Using Pulse Oximetry. Frank LH, Bradshaw E, PPHN from cyanotic CHD, the clinician must pay careful attention to patient
Beckman R, Mahle WT, Martin GR. J Pediatr. history; familial occurrence of CHD; and risk factors for PPHN, including sepsis,
2013;162(3):445–453
perinatal asphyxia, intrauterine growth restriction, and maternal medications (eg,
Diagnosis and Treatment of Pulmonary
selective serotonin reuptake inhibitors, indomethacin) (Table 1).
Hypertension in Infancy. Steinhorn RH. Early
Hum Dev. 2013;89(11):865–874 On physical examination, the clinician should auscultate for heart murmurs,
Role of Pulse Oximetry in Examining compare upper and lower extremity pulses, assess for hypotension associated
Newborns for Congenital Heart Disease: with the patient’s cyanosis, and examine the cardiac silhouette on chest radiog-
A Scientific Statement from the AHA and
raphy. For many years, the “hyperoxia test” was used to distinguish the 3 major
AAP. Mahle WT, Newburger JW, Matherne GP,
et al; American Heart Association Congenital causes of cyanosis. Arterial blood gases were obtained before and after infants
Heart Defects Committee of the Council on were placed in 100% oxygen. If an initially low PaO2 rose to equal to or greater than
Cardiovascular Disease in the Young, Council 150 mm Hg following hyperoxia, the likely diagnosis was respiratory disease.
on Cardiovascular Nursing, and
Interdisciplinary Council on Quality of Care Nonresponders with either cyanotic CHD or PPHN were subsequently hyper-
and Outcomes Research; American Academy ventilated. Decreased PaCO2 and alkalosis would dilate the pulmonary vasculature
of Pediatrics Section on Cardiology and in PPHN, resulting in increased PaO2, which would not be the case in cyanotic
Cardiac Surgery; Committee on Fetus and
Newborn. Pediatrics 2009;124(2):823–836 CHD. Recently, echocardiography with color Doppler directional flow has sup-
Persistent Pulmonary Hypertension of the planted the hyperventilation phase of the hyperoxia test. However, use of blood
Newborn: Advances in Diagnosis and gases is limited by the technical difficulty in obtaining them by arterial puncture; a
Treatment. Jain A, McNamara PJ. Semin Fetal falsely low PaO2 may result when an infant’s crying leads to increased right-to-left
Neonatal Med. 2015;20(4):262–271
shunting. Also, changes in clinical status may not be evident during the interval
Meconium Stained Fluid: Approach to the
Mother and the Baby. Welsh NC, Fanaroff JM. between the 2 samplings. Consequently, the routine introduction of pulse
Clin Perinatol. 2007;34(4):653–665 oximetry into the care of neonates has had a significant impact.

402 Pediatrics in Review


ductal patency in CHD lesions dependent on it for oxygenation,
TABLE 1. Disorders Associated with Persistent achievement and maintenance of higher saturations are readily
Pulmonary Hypertension of the noted. In PPHN associated with meconium aspiration syn-
Newborn drome, once the shunting resolves, the effects of parenchymal
lung disease may persist for days to weeks. Consequently, every
• Idiopathic
effort should be made to minimize barotrauma through
• Meconium aspiration syndrome “gentler” ventilation, use of high-frequency oscillatory ventila-
• Respiratory distress syndrome tion to optimize lung volumes, and tolerance of lower pH (7.25-
• Transient tachypnea of the newborn 7.29) and higher PaCO2 (50-60 mm Hg) in arterial blood.
Aside from its use in the early evaluation of cyanotic
• Pneumonia/sepsis
CHD, pulse oximetry can also play a key role in the detection
• Lung hypoplasia
of critical congenital heart disease (CCHD) in asymptomatic
• Congenital diaphragmatic hernia infants. CCHD, defined as lesions that require intervention
• Medications: Indomethacin, selective serotonin reuptake inhibitors in the first postnatal year for survival, account for approx-
• Other imately 25% of CHD cases, but the condition may not be
evident initially because of incomplete transition from the
- Alveolar capillary dysplasia
fetal circulation (ie, elevated PVR and/or continued patency
- Surfactant protein B deficiency of the ductus arteriosus). Heart lesions with ductal-dependent
- ABCA3 surfactant deficiency pulmonary or systemic blood flow are most prominent in
- Pulmonary lymphangiectasia this category (Table 2). Because fetal ultrasonography
detects only about 50% of CCHDs, particularly lesions with
- Congenital lobar emphysema
- Cystic adenomatoid malformation

Adapted from Steinhorn R. Diagnosis and treatment of pulmonary


hypertension in infancy. Early Human Dev. 2013;89(11):865-74.9, with
TABLE 2. Critical Congenital Heart Disease
permission from Elsevier. Defects
• Left-sided obstructive lesions

Hypoplastic left heart syndrome


A pulse oximeter uses 2 diodes emitting light at 640 nm
(red) and 940 nm (infrared), which are selectively absorbed Interrupted aortic arch

by oxyhemoglobin and deoxyhemoglobin, respectively. Mea- Critical coarctation of the aorta


surement of red and infrared light emerging from the tissue Critical aortic stenosis
and calculation of their ratio are used to determine the
• Right-sided obstructive lesions
oxygen saturation. Because visual recognition of cyanosis
Pulmonary atresia with intact septum
requires the presence of equal to or greater than 5 g/dL of
deoxyhemoglobin, pulse oximetry allows detection of more Tricuspid atresia
subtle levels of desaturation. In addition, placement of 1 Critical pulmonary stenosis
pulse oximeter probe on a preductal site (eg, right hand) and Tetralogy of Fallot
a second on a postductal site (eg, either foot) allows detection of
• Mixing lesions
ductal shunting, which occurs in 90% of cases of PPHN.
With a threshold pre-to-postductal saturation difference of Total anomalous pulmonary venous return

equal to or greater than 10%, oximetry can be used initially at Transposition of the great arteries
diagnosis as well as to detect onset of shunting with patient Truncus arteriosus communis
agitation or clinical interventions (eg, suctioning). Unlike
Reprinted from Bruno CJ, Havranek T. Screening for critical congenital
the labile oxygenation of the infant with PPHN, patients
heart disease in newborns. Adv Pediatr. 2015;62(1):211-226, with
with cyanotic CHD have fixed low oxygen saturations. How- permission from Elsevier.
ever, once prostaglandin E1 infusion is initiated to maintain

Vol. 37 No. 9 SEPTEMBER 2016 403


single-ventricle configurations, its overall reliability is lim- of neonates who have false-positive screening results for
ited. Therefore, in the absence of effective screening, delays CCHD, echocardiography performed on these children
in diagnosis occur, leading to increased mortality, surgical may reveal other clinically relevant conditions, such as
morbidity, and neurodevelopmental disability. Echocardiog- pulmonary hypertension. On this basis, CCHD screen-
raphy would be the gold standard for detection, but use of ing is cost-effective; with $5 to $10 expended per child
this modality in every baby would be impractical and cost- screened, detecting 1 case of CCHD before clinical dete-
ineffective. As a result, newborn screening with pulse rioration offsets the cost of screening approximately 2,000
oximetry has become the standard in an increasing number healthy newborns. Lesions that are the most difficult to
of states. Recently, screening has expanded from pulse detect include aortic outflow obstruction, which often presents
oximetry on a single site to 2 measurements: one preductal later compared to other CCHDs. Further studies are required
and the second postductal. As shown in the algorithm to determine the proper timing, if any, for CCHD screening in
(Figure), a positive result of screening when performed at the NICU, where pulse oximetry is already being used for
more than 24 hours of age consists of a single reading of less regular monitoring and other coexisting conditions (eg, pre-
than 90% on any site OR 3 consecutive hourly readings of maturity, sepsis, and primary lung disease) and can complicate
either 90% to 94% on any site or a difference of greater than analysis.
3% between upper and lower extremities. Pulse oximetry has had a dramatic impact on neonatal
This methodology for CCHD screening provides sensi- care in the: 1) detection of subtle cyanosis (<5 g/dL deoxy-
tivity between 75% and 80%, specificity greater than 99%, hemoglobin), 2) differentiation of CHD and PPHN in the
positive predictive value of 26%, and negative predictive infant with acute cyanosis, and 3) detection of CCHD in the
value of greater than 99%. Even with the fewer than 1% asymptomatic newborn.

Figure Pulse oximetry screening protocol.


Reprinted from the public domain from the U.S.
Centers for Disease Control and Prevention.
Available at http://www.cdc.gov/ncbddd/
heartdefects/hcp.html, as adapted from Kemper
AR, Mahle WT, Martin GR, et al. Strategies for
implementing screening for critical congenital
heart disease. Pediatrics. 128(5). Available at: www.
pediatrics.org/cgi/content/full/128/5/e1259.

404 Pediatrics in Review


COMMENT: For those of us who felt like medieval tor- can’t yet prevent the occurrence of critical congenital heart
turers when jabbing the radial arteries of cyanotic neonates disease, at least we have a safe and efficient tool for iden-
in a search for that arterial blood flow that seemed stub- tifying affected infants before they suffer irreversible harm,
bornly resistant to our desperate efforts, the advent of pulse which is progress indeed!
oximetry has been akin to the alchemist’s dream of turning
lead into gold. How much easier for us, and of course, more – Henry M. Adam, MD
importantly, how much nicer for the children. Further, if we Associate Editor, In Brief

Parent Resources from the AAP at HealthyChildren.org


Pulse Oximetry and the Neonate
• https://www.healthychildren.org/English/ages-stages/baby/Pages/Newborn-Screening-Tests.aspx
• Spanish: https://www.healthychildren.org/spanish/ages-stages/baby/paginas/newborn-screening-tests.aspx
• https://www.healthychildren.org/English/ages-stages/baby/preemie/Pages/When-Baby-Needs-Oxygen-At-Home.aspx
• Spanish: https://www.healthychildren.org/spanish/ages-stages/baby/preemie/paginas/when-baby-needs-oxygen-at-home.aspx

CME Quiz Corrections


An error was found in the CME quiz for the December 2015 article “Pain and Symptom Management in Pediatric
Palliative Care” (Komatz K, Carter B. Pediatrics in Review. 2015;36(12):257–534, doi: 10.1542/pir.36-12-527). The correct
answer for Question 4 should be “C. Ondansetron,” with the following rationale: “Ondansetron, a serotonin 5-HT3
receptor antagonist, may be used in children older than 4 years to prevent postoperative nausea and vomiting, or that
associated with chemotherapy.” The quiz has been corrected. A correction notice has been posted with the online version
of the article. The journal regrets the error.
In the CME quiz for the August 2016 review “Inflammatory Bowel Disease” (Shapiro JM, Subedi S, LeLeiko NS.
Pediatrics in Review. 2016;37(8):337–347, DOI: 10.1542/pir.2015-0110), the correct answer for Question 3 is “A.
Endoscopy.” The quiz has been corrected. A correction notice has been posted with the online version of the article.
The journal regrets the error.

ANSWER KEY FOR SEPTEMBER 2016 PEDIATRICS IN REVIEW:


Acid-Base Disorders: 1. A; 2. B; 3. E; 4. C; 5. B.
Evaluation and Initial Management of Hypopituitarism: 1. A; 2. E; 3. D; 4. D; 5. E.
Chronic and Recurrent Abdominal Pain: 1. B; 2. B; 3. B; 4. D; 5. C.

Vol. 37 No. 9 SEPTEMBER 2016 405


Cutting-Edge Resources
on Infectious Disease!
R Book® Atlas of Pediatric Infectious
Red
D
Diseases, 3rd Edition
Ed
Editor
Carol J. Baker, MD, FAAP
Ca
T
This best-selling resource aids in the diagnosis and treatment of more
than
t 100 of the most commonly seen pediatric infectious diseases.
New! Includes
I more than 1,200 full-color images to help you with disease
recognition.
r

Effectively problem-solve emerging


infectious diseases with these popular
resources:

New
4th
Edition!

®
PREP : ID
Save the Date! An Update of Pediatric Infectious Diseases and
Antimicrobial Therapy
+VMZ á%BMMBT 5FYBTá8FTUJO(BMMFSJB)PUFM
For a full listing of CME live activities, visit TIPQBBQPSHMJWFBDUJWJUJFT

Order today! Online at shop.aap.org ‡ Phone at 888/227-1770 toll-free from 7:30 am to 5:00 pm CT
AD250
Spoon Nails and Short, Brittle Hair in a
3-year-old Boy
Eric P. Sorensen, MD,* Wynnis L. Tom, MD†
*School of Medicine, University of California, San Diego, La Jolla, CA.

Departments of Dermatology and Pediatrics, University of California,
San Diego and Rady Children’s Hospital, San Diego, CA.

PRESENTATION

A 3-year-old boy presents for evaluation of toenails that have been abnormally
shaped since infancy and areas of hair on the scalp that are of a coarser texture
and that “never grow long at all.” There is no history of chemical or other insult to
the hair or nails. He has no known health problems and has been meeting growth
and developmental milestones appropriately. His pediatrician recommended a
trial of sulfur sulfacetamide shampoo to the scalp, but there was no improve-
ment. His older sister had similar hair findings that resolved by age 1 year. The
boy’s mother is concerned if there might be nutritional or other health concerns
because his condition has not improved. There is no other family history of hair or
nail abnormalities.
On physical examination, the well-developed and well-nourished boy has
appropriate behavior for age. All of his toenails are thin and depressed in a concave
manner (Fig 1), but all fingernails are unaffected. His hair is short, coarse, and
wiry predominately on the occipital scalp (Fig 2). Hair in all other areas appears
normal. The thyroid gland is not enlarged and the distal extremities are well
perfused, with no clubbing or cyanosis. Several strands of hair from the affected
area appear under microscopic examination in Figure 3.

DISCUSSION

Findings on physical examination and microscopic hair examination suggest the


diagnosis of koilonychia in the setting of monilethrix. According to Stone and
Maberry, causes of koilonychia (thin, concave-shaped nails) can be divided into 3
major categories: idiopathic, acquired, and hereditary in association with other ec-
todermal defects. In idiopathic cases, koilonychia is seen in an otherwise healthy
infant, and the condition tends to resolve on its own. In acquired cases, koilonychia
is often related to systemic diseases such as iron deficiency anemia and thyroid
AUTHOR DISCLOSURE Mr Sorensen and disease. Interestingly, in children with iron deficiency, koilonychia often
Dr Tom have disclosed no financial presents before significant anemia develops. Therefore, early detection via this
relationships relevant to this article. This
clinical sign may help prevent progression of iron deficiency to its more severe
commentary does contain a discussion of
an unapproved/investigative use of manifestations, such as neurodevelopmental defects and decreased immune
a commercial product/device. function. Koilonychia has been described in both hyperthyroidism (5% of

e38 Pediatrics in Review


Monilethrix is predominantly an autosomal dominant
condition, although autosomal recessive mutations have
been documented. The basic hair keratin genes (KRT81,
KRT83, and KRT86), along with the type II trichocyte keratin
genes (hHb1, hHb3, and hHb6), have been implicated in the
autosomal dominant form. Of note, mutations in hHb1 may
produce a less severe phenotype than with the other type II
keratin genes. Mutations in the desmoglein 4 (DSG4) gene,
which encodes for a protein in the desmosomal bridge
structures that help cells attach to each other, have been
found in autosomal recessive cases. The aberrancy in moni-
lethrix appears to be primarily attributable to cells in the
hair shaft cortex having dysfunction of the intracellular
cytoskeleton and cell-cell adhesion.

Differential Diagnosis
Figure 1. Thin toenails with concave curvature. Other hair shaft diseases may mimic monilethrix in appear-
ance, with the same sparse, short, dry, and fragile hair at an
cases) and hypothyroidism. Iron and thyroid studies should
early age. However, examination with light microscopy can
be considered in patients with persistent, acquired koilo-
allow differentiation, with electron microscopy used for
nychia. Correction of the systemic problem resolves the nail
further evaluation, if needed. Trichorrhexis nodosa is
changes. Other, less common systemic causes include sys-
a common congenital or acquired hair shaft disorder with
temic lupus erythematosus, hemochromatosis, and Raynaud
distinct points of splintering like two opposing broomsticks.
disease. Performing a thorough review of systems and physical
Pili torti is a hereditary hair shaft disorder characterized by
examination is important in determining if these conditions
light-colored fragile hair with a twisted appearance that can
warrant further consideration.
occur alone or as a manifestation of a syndrome, such as
The clinician should also consider the congenital ectoder-
Menkes syndrome. Wooly hair syndrome is characterized by
mal defect disorders as other potential causes of koilonychia,
tight, curly hair and has a variable inheritance pattern but
particularly if the condition is of early onset. Ectodermal-
is normally present from birth. Trichothiodystrophy refers
derived structures include the hair, teeth, nails, sweat glands,
to the sulfur-deficient brittle hair that gives a tiger-tail
cranial-facial structure, and digits. Among the congenital dis-
orders that may include koilonychia are monilethrix, palmo-
plantar keratodermas, and the ectodermal dysplasia syndromes,
where 2 or more ectodermal-derived structures are affected.
Monilethrix is a rare, heritable disorder with onset at any age,
although it predominantly begins in early childhood. Hair shafts
have areas of normal growth separated by areas of narrowing,
called nodes and internodes, respectively. Microscopic examina-
tion demonstrates regularly spaced spindlelike segments, giving
the monilethrix hair a characteristic string-of-beads appearance.
Internodes typically form soon after birth, at intervals of 0.7 to
1.0 mm. This thinning causes the hair to break at a shorter length
than normal, forming short, sparse, coarse, and wiry hairs. The
nape and occipital regions are most commonly affected, but hair
over the entire body has the potential to be involved. In more
severe cases, the condition can progress to a scarring alopecia.
Frequently associated with monilethrix are koilonychia and ker-
atosis pilaris, a common, benign condition of small, rough, follicle-
centered papules. Less commonly associated findings are other
dental and nail anomalies, juvenile cataracts, and neurologic dis-
orders, such as cognitive impairment, schizophrenia, and epilepsy. Figure 2. A transition from normal hair to coarse, wiry hair at the occiput.

Vol. 37 No. 9 SEPTEMBER 2016 e39


Figure 3. A) Examination of affected and
unaffected hairs with light microscopy at 4
magnification. B) An affected hair at 40
magnification.

appearance under polarizing microscopy, seen in a group options were discussed, but none were chosen because data
of autosomal recessive genetic disorders with ichthyosis, on their usage were limited. Biotin 2.5 mg was started as
short stature, neurologic impairment, and photosensitivity. a supplement to potentially help strengthen the hair and
Chemical or heat insult may also lead to brittle hair, as can nails, based on use for other hair shaft disorders and for nail
severe malnutrition (kwashiorkor, marasmus), but the other- fragility, along with its low risk profile.
wise healthy boy in this case lacks such a history. Iron deficiency
and thyroid disorders do not tend to cause patterned changes
along the hair shaft, although they can lead to dry, thin hair Summary
and diffuse shedding. • The causes of koilonychia (spoon nails) may be idiopathic,
Of note, normal hair may have an appearance similar to that acquired, hereditary, and in association with other ectodermal
defects. Acquired causes include iron deficiency anemia and
of monilethrix under light microscopy as an artifact of the fixing
thyroid disease, and hereditary causes include monilethrix.
process. This phenomenon, called pseudomonilethrix, may be Performing a thorough review of systems and physical
caused by trauma from tweezers or forceps and by compressing examination helps determine which conditions are probable and
overlapping hairs between glass slides. It can be avoided by warrant further evaluation.
cutting the hair shaft instead of plucking and minimizing • Monilethrix is a predominantly autosomal dominant hair shaft
friction between the two glass slides when preparing the sample. disorder characterized by sparse, short, dry, and fragile hair.
Koilonychia in this setting does not have other systemic implications.
Management • The distinguishing characteristic of the monilethrix hair is its beaded
The prognosis of monilethrix is variable; in some cases, the appearance, caused by regularly spaced deformation of cortical hair
findings resolve spontaneously. Several accounts report an shaft cells and visible under light and electron microscopy.

improvement with hormonal changes following the onset of • Literature supporting treatment options is limited to only a few
case reports, although some patients note gradual self-resolution
adolescence or pregnancy. However, in one case, hormonal
and others have improvement with puberty.
treatment only gave temporary improvement.
Currently there is no specific treatment for monilethrix.
Positive responses have been reported with topical minox-
idil and several oral medications (corticosteroids, griseo-
Suggested Readings
fulvin, N-acetyl cysteine, and the retinoids acitretin and
Farooq M, Ito M, Naito M, Shimomura Y. A case of monilethrix caused
etretinate). However, none have been consistently effective,
by novel compound heterozygous mutations in the desmoglein 4
and the risks versus benefits must be weighed when consid- (DSG4) gene. Br J Dermatol. 2011;165(2):425–431
ering treatment. Some reports claim an exacerbation of moni- Gebhardt M, Fischer T, Claussen U, Wollina U, Elsner P. Monilethrix–
lethrix with trauma, so minimizing trauma and damage to the improvement by hormonal influences? Pediatr Dermatol. 1999;
hair may help. Genetic testing for mutations does not tend to 16(4):297–300

alter management but may inform future/familial risk. Ito M, Hashimoto K, Katsuumi K, Sato Y. Pathogenesis of monilethrix:
computer stereography and electron microscopy. J Invest Dermatol.
1990;95(2):186–194
Patient Course
Leitner C, Cheung S, de Berker D. Pitfalls and pearls in the diagnosis of
Examination of hair samples from affected and nonaffected monilethrix. Pediatr Dermatol. 2013;30(5):633–635
areas for the boy in this case confirmed the diagnosis of Rasmussen J. Hair loss in children. Pediatr Rev. 1981;3(3):85–90
monilethrix. Laboratory studies showed normal thyrotropin, Stone OJ, Maberry JD. Spoon nails and clubbing: review and possible
free thyroxine, and complete blood cell count values. Treatment structural mechanisms. Tex Med. 1965;61:620–627

e40 Pediatrics in Review

You might also like